You are on page 1of 39

1

CHAPTER

Units and Measurements

Year Wise Number of Questions Analysis (2023-2014)

5
Number of Questions

0
2023 2022 2021 2020 2020 2019 2018 2017 2016 I 2016 II 2015 2015 Re 2014
Covid
Years

Topicwise Number of Questions Analysis (2023-2014)

12
6 5 6 2
11

10
Error, Measurement & Significant Figure

9
Application of Dimensional Formula
Dimension & Dimensional Formula
Number of Questions

6
Measuring Instruments

Topics
Weak Nuclear Force
PHYSICAL WORLD ™ The weak nuclear force is not as weak as the gravitational
What is Science? force but much weaker than strong nuclear force.
™ Science is a systematic attempt to understand natural phenomena ™ The weak nuclear force appears only in certain nuclear
in as much detail and depth as possible, and use the knowledge processes b-decay of a nucleus.
so gained to Predict, Modify and Control phenomena. ™ The range of weak nuclear force is exceedingly small of the
™ The word ‘Science’ originates from the latin verb ‘Scientia’ order 10–16m.
meaning ‘to know’. ™ In b-decay the nucleus emits an electron and an uncharged
™ The Sanskrit word ‘Vijnan’ and the Arabic word ‘Ilm’ particle called anti-neutrino.
convey same meaning, namely ‘Knowledge’.
Key Note
What is Physics?
™ Physics is a study of the basic laws of nature and its P Range of gravitational Force > Range of Electromagnetic
manifestation in different natural phenomena. Force > Range of Nculear force.
™ Physics comes from a Greek word “Fusis” signifies ‘nature’.
P Strength of Nuclear Force > Strength of Electromagnetic
™ The Sanskrit word ‘Bhautiki’ convey similar meaning Force > Strength of Gravitational Force.
‘to the study of physical world’.
Fundamental Forces in Nature Table: Fundamental Force of Nature
There are four fundamental forces in nature.
Relative Operatres Mediating
Gravitational Force Name Range
Strength Among Particle
™ Gravitational force is weakest force and conservative in Gravitational Force 10–39 Infinite All objects in Graviton
nature. the universe
™ It is the force of mutual attraction between any two objects by Weak Nuclear Force 10–13 Very short, Some Boson
Sub- elementary
virtue of their masses. nuclear particles,
™ It is a universal force and always attractive in nature. size Particularly
™ It plays a key role in the large scale phenomena of universe (∼10–16m) electron and
antineutrino
such as formation and evolution of stars, galaxies and galactic
Electromagnetic 10–2 Infinite Charges Photon
clusters. Particles
™ The gravitational force is appreciable only when at least one Strong Nuclear Force 1 Short Nucleons, Gluon
of the two bodies has a large mass. Nuclear Havier
Electromagnetic Force size elementary
(∼10–15m) Particles
™ Electromagnetic force is the force between charge particles.
™ They are attractive as well as repulsive in nature.
PHYSICAL QUANTITIES
™ When charges are in motion, they produce magnetic field
giving rise to a force on a moving charge. All quantities that can be measured are called physical quantities.
™ It is quite strong compared to gravity. e.g., length, mass, force, work done, etc. In physics we study
™ When charges are at rest, the force is given by coulomb’s law. about physical quantities and their inter relationship.
™ Electric and magnetic effects are in general inseparable; There are two types of physical quantities
hence the name electromagnetic force. 1. Fundamental Quantity: Physical quantities which cannot
™ Like the gravitational force, electromagnetic force act over be expressed in terms of any other physical quantities are
large distances and does not need any intervening medium. called fundamental physical quantities.
™ For example electric force between two protons is 1036 times e.g., length, mass, time, temperature etc.
the gravitational force between them, for a certain distance. 2. Derived Quantity: Physical Quantities which are derived
Strong Nuclear Force from fundamental quantities are called derived quantities.
™ It is independent of charge. e.g., Area, density, force etc.
™ The strong nuclear force binds protons and neutrons in a nucleus.
It is evident that without some attractive force, a nucleus will be MEASUREMENT
unstable due to electric repulsion between protons. Measurement is the comparison of a physical quantity with a
™ The strong nuclear force is the strongest of all fundamental standard of the same physical quantity.
forces. Different countries followed different standards.
™ Recent developments have however indicated that protons and
neutrons are composed of still more elementary constituents
Units of Measurement of Physical Quantity
A fixed measurement chosen as a standard of measurement to
called quarks.
measure a physical quantity is called a Unit.
™ It’s range is extremely small of the order nuclear dimensions
(10–15m). To measure a physical quantity means to determine the number of
™ It is responsible for the stability of nuclei.
times its standard unit is contained in that physical quantity.

2 P NEET (XI) Module-1 PHYSICS


W
A standard Unit is necessary for the sake of 2. CGS or Gaussian system: In this system the fundamental
1. Accuracy, 2. Convenience, quantities are length, mass and time and their respective units
3. Uniformity and 4. Equal justice to all. are centimetre (cm), gram (g) and second (s).
The standard unit chosen should have the following characteristics. 3. MKS system: In this system also the fundamental quantities
1. Consistency or invariability are length, mass and time but their fundamental units are
metre (m), kilogram (kg) and second (s) respectively.
2. Availability
Units of some fundamental physical quantities in different systems
3. Imperishability Permanency
4. Convenience and acceptability. Type of physical Physical Quantity System
5. Reproducibility Quantity
Based on the dependency units are classified in two types: CGS MKS FPS
1. Fundamental unit: The unit used to measure the fundamental Length cm m ft
quantity is called fundamental unit. Fundamental Mass g kg lb
e.g., Metre for length, kilogram for mass etc. Time s s s
2. Derived unit: The unit used to measure the derived quantity 4. International system (SI) of units: This system is
is called derived unit. modification over the MKS system. Besides the three base
e.g., Metre2 for area, gram Centimeter–3 for density etc. units of MKS system four fundamental and two supplementary
Systems of Units units are also included in this system.
There are four systems of units Based on SI there are three categories of physical quantities:
1. FPS or British Engineering system: In this system length, 1. 7 Fundamental quantities
mass and time are taken as fundamental quantities and their 2. 2 Supplementary quantities
base units are foot (ft), pound (lb) and second (s) respectively. 3. Derived quantities

FUNDAMENTAL QUANTITIES AND THEIR S.I. UNITS


There are seven fundamental quantities and two supplementary quantities in S.I. system. The names and units with symbols are given below:

Classification Physical Quantity Unit Symbol Definition

1
Length (m) Metre m The distance traveled by light in vacuum in second
is called 1 metre. 299,792, 458

The mass of a cylinder made of platinum-iridium alloy kept at


Mass (kg) Kilogram kg
International Bureau of Weights and Measures is defined as 1 kilogram.
The second is the duration of 9,192,631,770 periods of the radiation
Time (s) Second s corresponding to the transition between the two hyperfine levels of
the ground state of the cesium-133 atom.
If equal currents are maintained in the two parallel infinitely long
wires of negligible cross-section, so that the force between them is
Fundamental Electric Current (I) Ampere A
2 × 10–7 newton per metre of the wires, the current in any of the
Quantity wires is called 1 Ampere.

Thermodynamic 1
Kelvin K The fraction of the thermodynamic temperature of triple
Temperature (K) 273.16
point of water is called 1 Kelvin.
1 candela is the luminous intensity of a blackbody of surface area
1
Luminous Intensity (cd) Candela cd m 2 placed at the temperature of freezing platinum and at a
600,000
pressure of 101,325 N/m2, in the direction perpendicular to its surface.
The mole is the amount of a substance that contains as many elementary
Amount of substance (mole) Mole mol
entities as there are number of atoms in 0.012 kg of carbon-12.
Supplementary 1. Plane angle (radian) Radian Rad Angle = arc/radius, θ = l/r
Quantity
2. Solid Angle (steradian) Steradian sr W = Area/ (Radius)2

Units and Measurements 3


Some Special Units for Length 1 L = 1000 cm3 = 10–3 m3
Angstrom (Å) = 10 m = 10 cm
–10 –8 1kWh = 36 × 105 J
Nanometre (nm) = 10–9 m = 10 Å 1 HP = 746 W
Fermi = 10–15 m π
1 degree = rad.
Micron = 10–6 m 180
X-ray unit = 10–13 m 1 kg-wt = 9.8 N
1 Astonomical Unit (A.U.) = Mean distance between sun & earth 1 tesla (T) = 104 gauss
= 1.496×1011 m 1 weber = 108 maxwell
1 Light year = Distance travelled by light in one Some Physical Constants and their Values
year in vacuum or air having speed 1 amu = 1.67 × 10–27 kg = 931 MeV
3 × 108 m/s.
1 atm = 76 cm of Hg = 1.013 × 105 Pa
= 9.46 × 1015 m Avagadro number (N) = 6.023 × 1023
Parsec = 3.26 light years = 30.84 × 1015 m Permittivity of free space (ε0) = 8.854 × 10–12 F–m–1
Bohr radius = 0.5 × 10–10 m Permeability of free space (m0) = 4p × 10–7 H–m–1
Mile = 1.6 km Joule’s constant (J) = 4.186 J Cal–1
Some Special Units for Mass Planck’s constant (h) = 6.62 × 10–34 Js
Quintal = 100 kg Rydberg’s constant (R) = 1.0974 × 107 m–1
Metric ton = 1000 kg Boltzmann’s constant (K) = 1.38 × 10–23 JK–1
Atomic mass unit (a.m.u) = 1.67 × 10–27 kg Stefan’s constant (s) = 5.67 × 10–8 Wm–2 K–4
Chandra Sekhar Limit = 1.4 times mass of the sun Universal gas constant (R) = 8.314 J mol–1 K–1
= 1.98 cal mol–1 K–1
Some Special Units for Time Wien’s constant (b) = 2.93 × 10–3 mK
Shake = 10–8 second
DIMENSIONS AND
Abbreviations for Multiples and Sub Multiples
Macro Prefixes:
DIMENSIONAL FORMULA
Multiplier Symbol Name All the physical quantities of interest can be derived from the base
101 da Deca quantities. The power (exponent) of base quantity that enters into
102 h Hecto the expression of a physical quantity, is called the dimension of
103 K Kilo the quantity in that base. To make it clear, consider the physical
quantity force.
106 M Mega
Force = mass × acceleration
109 G Giga
1012 T Tera Velocity
= mass ×
1015 P Peta time
length/time
Micro Prefixes: = mass ×
time
Multiplier Symbol Name = mass × length × (time)–2
10–1 d deci So the dimensions of force are 1 in mass, 1 in length and –2 in
10–2 c centi time.
10–3 m milli Similarly energy has dimensions, 1 in mass, 2 in length and –2
10–6 m micro in time.
10–9 n nano Such an expression for a physical quantity in terms of base
10–12 p pico quantities is called dimensional formula.
10–15 f femto Based on dimensions Physical quantity can be further divided
in to four types:
Some Important Conversions 1. Dimensional Constants: The physical quantities which have
1000m 5 dimensions and have a fixed value are called dimensional
1 km/h = = ms −1 constants.
3600s 18
e.g., Gravitational Constant (G), Planck’s Constant (h), Universal
1 newton = 105 dyne gas constant (R), Velocity of light in vacuum (c) etc.,
1 joule = 107erg 2. Dimensionless Quantities: Dimensionless quantities are
1 calorie = 4.18 Joule those which do not have dimensions but have a fixed value.
1 electron Volt = 1.6 × 10–19 Joule (i) Dimensionless quantities without units.
1 gcm–3 = 1000 kgm–3 e.g., Pure numbers, angle etc.

4 P NEET (XI) Module-1 PHYSICS


W
(ii) Dimensionless quantities with units. 1
e.g., Angular displacement (radian), Joule’s constant x = x0 + v0 t + at 2
2
(joule/calorie) etc.
3. Dimensional Variables: Dimensional variables are those Check the dimensional consistency of this equation.
physical quantities which have dimensions and do not have Sol. The dimension of the various terms are
fixed value. [x] = [L]
e.g., velocity, acceleration, force, work, power... etc. [x0] = [L]
4. Dimensionless Variables: Dimensionless variables are those [v0t] = [LT–1][T] = [L]
physical quantities which do not have dimensions and do not 1 2 
have fixed value.  at  = [LT–2][T2] = [L]
e.g., Specific gravity, refractive index, Coefficient of friction, 2 
Poisson’s Ratio etc. Since the dimensions of all the terms are same. Hence the
given equation is dimensionally correct.
DIMENSIONAL EQUATION
Whenever the dimension of a physical quantity is equated with its Train Your Brain
dimensional formula, we get a dimensional equation.
e.g., E = [M1L2T–2].
Dimension Example 1: Check the accuracy of the relation T = 2π
L
Dimensions of a physical quantity are the powers to which the g
fundamental units are to be raised to obtain one unit of that for a simple pendulum using dimensional analysis.
quantity. Sol. The dimensions of LHS = the dimension of T = [M0L0T1]
Dimensional Formula  dim.of length 
1/2

An expression showing the powers to which the fundamental units The dimensions of RHS = 
 dim.of acc n 
are to be raised to obtain one unit of the derived quantity is called  
Dimensional formula of that quantity. (Q 2p is a dimensionless const.)
In general the dimensional formula of a quantity can be written  L 
1/2
x y z
as [M L T ]. Here x, y, z are dimensions of mass, length and time =  −2 
= (T 2 )1/2
= (T = ) [ M 0 L0T 1 ]
respectively.  LT 
So Dimensions of LHS = Dimensions of RHS
Principle of Homogeneity of Dimension Example 2: Find dimensions of constant a and b in equation
This principle states that the dimensions of all the terms in a
 a 
physical expression should be same. For example, in the physical  P + 2  (V − b) =RT
1 1  V 
expression s = ut + at 2 , the dimensions of s, ut and at 2 all
2 2 where P is pressure and V is volume, R is universal gas
are same.
constant and T is temperature.
Note: The physical quantities separated by the symbols +,–, =, >,<
Sol. We can add and subtract only like quantities.
etc., have the same dimensions.
a
So, Dimensions of P = Dimension of 2
V
USES OF DIMENSIONAL ANALYSIS ∴ Dimensions of a = Dimension of P × Dimension of V2
(i) Dimensional formulae help us to understand the physical Dimension of a = [ML–1T–2] [L3]2
behaviour of a quantity. Two different Physical quantities Dimension of a = [M1L5T–2]
having different dimensions can not be added or subtracted. and Dimensions of b = Dimension of V
Thus we can not add or Subtract displacement to the velocity. So, Dimension of b = [L3]
Thus we must remember that Example 3: Find the dimension of (a/b) in the equation:
 It A+B and A – B are meaning full then and A and B must a − t2
P=
have same dimension and same nature. bx
 If A = B, then A and B. have some dimension and same where P is pressure, x is distance and t is time.
nature. Sol. Using principle of Homogeneity,
(ii) To check the dimensional correctness of a given physical Dimension of a
relation: It is based on principle of homogeneity, which states ∴ Dimension of P =
Dimension of b × Dimension of x
that a given physical relation is dimensionally correct if the
Dimension of a
dimensions of the various terms on either side of the relation = Dimension of P × Dimension of x
are the same. Dimension of b
Ex: The distance x travelled by a body in time t which starts   = [ML–1T–2] [L]
from the position x0 with initial velocity v0 and has uniform   = [M1L0T–2]
acceleration a, is given by:

Units and Measurements 5


Concept Application Train Your Brain

1. Find the dimension of physical quantity X in the Example 4: If P is the pressure of a gas and r is its density,
X then find the dimension of velocity in term of pressure and
equations, Force = .
Density density.
x  Sol. Method-I
2. The equation of a wave is given by Y= A sin ω  − k  .
v  [P] = [ML–1T–2]...(i)
where w is the angular velocity and v is the linear velocity. [r] = [ML–3]...(ii)
Find the dimension of k. Dividing eq. (i) by (ii)
3. In a book, the answer for a particular question is [Pr–1] = [L2T–2]
ma  2kl  ⇒ [LT–1] = [P1/2r–1/2] ⇒ [V] = [P1/2r–1/2]
expressed
= as b  1+  here m represents mass,
k  ma  Method-II
a represents accelerations, l represents length. then the v ∝ Pa rb
unit of b should be. v = kPa rb
[LT–1] = [ML–1T–2]a [ML–3]b
1. To establish a relation between different physical Comparing both side
quantities: If we know the various factors on which a 1 1
a = , b = – ⇒ [V] = [P1/2r–1/2]
physical quantity depends, then we can find a relation among 2 2
different factors by using principle of homogeneity.
2. To Convert the physical quantity from one system of unit
Ex: Let us find an expression for the time period T of a simple

to another system of unit: According to this magnitude of
pendulum. The time period T may possibly depend upon physical quantity remains same whatever system is used for
(i) Mass m of the bob of the pendulum measure
i.e., magnitude = numerical value × unit = constant
(ii) Length l of pendulum
\ nu = Constant
(iii) Acceleration due to gravity g at the place where the So, for two unit systems.
pendulum is suspended. n1u1 = n2u2
Sol. Let (i) T ∝ ma  (ii) T ∝ lb  (iii) T ∝ gc If a quantity in represented by [MaLbTc]
Combining all the three factors, we get Then, n1 [M1aL1bT1c] = n2 [M2aL2bT2c]
Where n1 and n2 are numerical values in both systems.
T ∝ malbgc
M1 and M2 are units of mass in system 1 and 2 respectively.
or T = Kmalbgc L1 and L2 are units of Length in system 1 and 2 respectively.
Where K is a dimensionless constant of proportionality. T1 and T2 are units of Time in system 1 and 2 respectively.
Writing down the dimensions on either side of equation,
we get Train Your Brain
[M0L0T1] = [Ma][Lb][LT–2]c = [MaLb + cT–2c]
Example 5: Find the value of 60J per min on a system that
Comparing power of M, L and T
has 100g, 100cm and 1min as the fundamental units.
a = 0, b + c = 0, 60 joule 60 joule
Sol. Here, P = = = 1watt
–2c = 1 min 60sec
1 The SI units of Power = [M1 L2 T–3]
⇒ c= −
2 SI New System of unit
1 n1 = 1 n2 = ?
⇒ b=–c=
2 M1 = 1 Kg = 1000 g M2 = 100 g
L1 = 1 m = 100 cm L2 = 100 cm
l
So, T = K T1 = 1 sec T2 = 1 min = 60 sec
g

6 P NEET (XI) Module-1 PHYSICS


W
Using, n1 u1 = n2 u2 Example 8: A calorie is a unit of heat or energy and it equals
1 2 −3
about 4.2 J, where 1 J = 1 kg m2/s2. Suppose we employ a
M  L  T  system of units in which the unit of mass equals a kg, the
n2 = n1  1   1   1 
 M 2   L 2   T2  unit of length equals b metre, the unit of time is g second.
Show that a calorie has a magnitude 4.2 a–1b–2g2 in terms of
1 2
1000 g  100 cm   1sec 
−3
the new units.
= 1     
 100 g  100 cm   60sec  Sol. 1 cal = 4.2 kg m2s–2
SI New system
n2 = 2.16 × 106
n1 = 4.2   n2 = ?
∴ 60 Joule/min = 2.16 × 106 new units of power. M1 = 1 kg M2 = a kg
Example 6: If the units of force, energy and velocity are L1 = 1 m L2 = b metre
20 N, 200 J and 5 ms–1, Find the units of Length, mass and T1 = 1 s T2 = g second
Dimensional formula of energy is [ML2T–2]
time.
1 2 −2
Sol. We have  M   L  T 
Now, n2 = n1  1   1   1 
[M L T–2] = 20N ...(i)  M 2   L2   T2 
[M L T ] = 200J
2 –2
...(ii) 1 2 −2
 1 kg  1 m  1 s  −1 −2 2

= 4.2    β m   γ s  = 4.2 α β γ
[L T–1] = 5ms–1...(iii)  α kg     
Dividing (ii) by (i),
[ML2 T −2 ] 200
=
[MLT −2 ] 20

or L = 10 m Concept Application
Putting the value of L in eq. (iii),
4. If the time period (T) of vibration of a liquid drop depends
[LT–1] = 5 on surface tension (S), radius (r) of the drop, and density
or 10T–1 = 5 (ρ) of the liquid, then find the expression of T.
1 5 5. If P represents radiation pressure, C represents the speed
or = of light, and Q represents radiation energy striking a unit
T 10
area per second, then non-zero integers, x, y, and z such
T = 2sec
then Px Qy Cz is dimensionless, find the value of x, y, and
Again eq. (i), z.
M × 10 × (2)–2 = 20 6. If velocity (v), force (F), and energy (E) are taken as
20 × 4 fundamental units, then find the dimensional formula
or M = for mass.
10
7. If force, acceleration and time are taken as fundamental
M = 8 kg
quantities, then find the dimensions of length?
Example 7: Convert 1 newton (SI unit of force) into dyne 8. If area (A), velocity (v) and density (ρ) are base units,
(CGS unit of force) then find the dimensional formula of force.
Sol. The dimensional equation of force is 9. The potential energy of a particle varies with distance x
[F] = [M1 L1T–2] A x
form a fixed origin as U = 2 , where A and B are
Therefore if n1, u1 and n2, u2 corresponds to SI & CGS x +B
unit respectively, then dimensional constants, then find the dimensional formula
1 1 −2 for AB.
M  L   T1 
n2u2 = n1u1 ⇒ n2 = n1  1   1    10. If the present units of length, time, and mass (m, s, kg)
 M 2   L2   T2  1
are changed to 100 m, 100s, and kg , then how will
 kg   m   s 
−2
the new unit of force change? 10
= 1      = 1× 1000 × 100 × 1 = 105
 g   cm   s  11. Convert 1MW power on a new system having basic
units of mass, length, and time as 10kg, 1dm, and 1min,
= 105 dyne = 1N respectively.

Units and Measurements 7


Key Note

P Full names of the units,even when they are named after a scientist should not be written with a capital letter.
e.g., newton, watt, ampere, metre.
P Units do not take plural form.
e.g., 10 kg but not 10 kgs, 20 W but not 20 Ws, 2A but not 2 As
P No full stop or punctuation mark should be used within or at the end of symbols for units.
e.g., 10 W but not 10 W.
P Powers are dimensionless
P sin q, eq, cos q, log q gives dimensionless value and in above expression q is dimensionless.
Dimensions of some Mathematical functions
dn y  y 
P  n = n  [dny treated as y only]
 dx   x 
 yds  = [ y x]
P ∫ 
P All trigonometric exponential and logarithmic functions are dimensions.

The Following is the list of some Physical Quantities with their Formula and Dimensional Formula
Dimensional
S. No. Physical Quantity Explanation or Formulae S.I.Unit
Formulae
1. Distance, Displacement, Wave Length, [M0 L1 T 0] m
Radius of gyration, Circumference,
Perimeter, Light year, Par-sec.
2. Mass [M1 L0 T 0] kg
3. Period of oscillation, Time, time total time [M0 L0 T1] s
constant T=
No. of oscillations
T = Capacitance × Resistance

1
4. Frequency Reciprocal of time period n = [M 0 L0 T –1] Hertz (Hz)
T
5. Area A = length × breadth [M0 L2 T 0] m2
6. Volume V = Length × breadth × height [M0 L3 T 0] m3
Mass
7. Density D= [M1 L–3 T 0] kgm–3
Volume

Mass
8. Linear density λ= [M1 L–1 T 0] kgm–1
Length

displacement
9. Speed, Velocity v= [M0 L1 T –1] ms–1
time

Change in Velocity
10. Acceleration a= [M 0 L1 T –2] ms–2
time
11. Linear Momentum P = mass × velocity [M1 L1 T –1] kgms–1
12. Impulse J = Force × time [M1 L1 T –1] Ns
13. Force F = Mass × acceleration [M1 L1 T –2] N

8 P NEET (XI) Module-1 PHYSICS


W
The Following is the list of some Physical Quantities with their Formula and Dimensional Formula
Dimensional
S. No. Physical Quantity Explanation or Formulae S.I.Unit
Formulae
14. Work, Energy, PE, KE, Strain Energy W = Force × displacement [M1 L2 T –2] J(or) N.m
1 2
P.E = mgh; KE = MV
2
1
SE = × Stress × Strain × Volume
2

Work
15. Power P= [M1 L2 T–3] Js–1 (or) watt
time
16. Pressure, Stress, Modulus of Elasticity Force [M1 L–1 T–2] Nm–2 (or) Pascal
(Y, n, k) pressure =
Area
Stress
Y=
Strain

Change in dimension
17. Strain [M0 L0 T0] No units
Original dimensions

Work
18. Strain energy density E= [M1 L–1 T–2] Jm–3
Volume

length of arc
19. Angular displacement θ= [M0 L0 T0] rad
radius

angular displacement
20. Angular Velocity ω= [M0 L0 T–1] rads–1
time

Change in angular velocity


21. Angular acceleration α= [M0 L0 T–2] rads–2
time
22. Angular momentum L = Linear momentum × arm [M1 L2 T–1] Js

energy
23. Planck’s constant h= [M1 L2 T–1] Js
frequency

24. Angular impulse Torque × time [M1 L2 T–1] Js


25. Torque τ = force × ⊥ distance [M L T ]
1 2 –2
Nm
26. Acceleration due to gravity (g) = weight [M L T ]
0 –2
ms–2 or Nkg–1
gravitational field strength g=
mass

F .d 2
27. Universal gravitational Constant G= [M–1 L3 T–2] Nm2 kg–2
M1.M 2

28. Moment of inertia I = MK2 [M1 L2 T0] kgm2

dv
29. Velocity gradient [M0 L0 T–1] s–1
dx
30. Surface Tension, Spring Constant surface energy force [M1 L0 T–2] Nm–1 or Jm–2
S= =
change in area length
force
K=
elongation

Units and Measurements 9


The Following is the list of some Physical Quantities with their Formula and Dimensional Formula
Dimensional
S. No. Physical Quantity Explanation or Formulae S.I.Unit
Formulae
tangential stress
31. Coefficient of Viscosity η= [M1 L–1 T–1] Pa s (or) Ns m–2
Velocity gradient

32. Gravitational Potential Gravitational field × distance [M0 L2 T–2] J.Kg–1

33. Heat energy msq [M1 L2 T–2] Joule

34. Temperature q [M0 L0 T0. q1] Kelvin (K)

dQ
35. Thermal Capacity = Mass × Sp. ht [M1 L2 T–2. q–1] JK–1

heat energy
36. Specific heat Capacity S (or)C = [M0 L2 T–2. q–1] Jkg–1 K–1
mass×temp.

heat energy
37. Latent heat (or) Calorific value L= [M0 L2 T–2] Jkg–1
mass
38. Water Equivalent W = MC [M1 L0 T0] kg
39. Coefficient of Thermal expansion a or b or g ask [q ]
–1
k–1

PV
40. Universal gas constant R= [M1 L2 T–2 q–1 mol–1] Jmol–1 K–1
nT

R
41. Gas constant (for 1 gram) r= [M0 L2 T–2 q–1 mol–1] Jkg–1 K–1
Mol.wt

R
42. Boltzman constant (for 1 Molecule) k= [M1 L2 T–2 q–1] JK–1 molecule–1
AvagadroNo.

W
43. Mechanical equivalent of heat J= [M0 L0 T0] no S.I. units
H

Q.d Js–1 m–1 K–1 (or)


44. Coeff of Thermal Conductivity K= [M1 L1 T–3 q–1]
A ∆θ.t Wm–1 K–1

dQ heat energy
45. Entropy = [M1 L2 T–2 q–1] JK–1
T temperature

∆E Js–1 m–2 K–4 (or)


46. Stefan’s Constant σ= [M1 L0 T–3 q–4]
A.∆T .θ 4 Wm–2 K–4

dθ temp × time
=R =
47. Thermal resistance  dQ  Heat [M–1 L–2 T3 q1] KsJ–1
 
 dt 

Change in temp d θ
48. Temperature gradient = [q L–1] Km–1
length dl

Change in pressure dp
49. Pressure gradient = [M1 L–2 T–2] Pascal m–1
length dl

10 P NEET (XI) Module-1 PHYSICS


W
The Following is the list of some Physical Quantities with their Formula and Dimensional Formula
Dimensional
S. No. Physical Quantity Explanation or Formulae S.I.Unit
Formulae
Energy ∆E
50. Solar constant = [M1 L0 T–3] Js–1m–2 (or) Wm–2
area × time .
AT

51. Enthalpy Heat. (DQ) [M1 L2 T–2] Joule

52. Pole strength M = I.L (or) [M0 L T0 A] Am


Magnetic Momement
Mag.Length

53. Magnetic Moment M = 2l × m = pole strength × length of magnet [M0 L2 T0 A] Am2

m
54. Magnetic intensity (or) Magnetising field H= [M0 L–1 T0 A] Am–1
4πd 2

M Magnetic Moment
55. Intensity of Magnetisation I
= = [M0 L–1 T0 A] Am–1
V Volume

56. Magnetic flux φ= B × A = (Magnetic induction × area) [M1 L2 T–2 A–1] Wb

φ Magnetic flux F Tesla (or) Wbm–2


57. Magnetic induction field strength B
= = = [M1 L0 T–2 A–1]
A area il (or) Na–1 m–1

4π.Fd 2
58. Magnetic permeability of free space µ0 = [M1 L1 T–2 A–2] Hm–1
m1.m2

I
59. Magnetic susceptibility χ= [M0 L0 T0] No units
H
60. Electric current I [M0 L0 T0 A.] A
61. Charge Q = Current × time [M L T .A]
0 0
C
62. Electric dipole moment P = Charge × distance [M L T .A]
0 0
Cm

Electric field strength (or) Electric field force


63. E= [M1 L T–3 A–1] NC–1
Intensity Charge

64. Electrical flux (fE) Electrical Intensity × area [M1 L3 T–3 A–1] Nm2C–1

Work
65. Electric potential (or) Potential difference V= [M1 L2 T–3 A–1] V
Charge

Pot.diff
66. Electrical resistance R= [M1 L2 T–3 A–2] W
Current

1 1 mho (or) siemen


67. Electrical conductance C= = [M–1 L–2 T3 A2]
R Resistance (S)

R. A
68. Specific resistances (or) Resistivity r (or) ρ= [M1 L3 T–3 A–2] Ohm m
l

1 Ohm–1 m–1 (or)


69. Electrical conductivity s= [M–1 L–3 T3 A2]
Resistivity siemen m–1

Units and Measurements 11


The Following is the list of some Physical Quantities with their Formula and Dimensional Formula
Dimensional
S. No. Physical Quantity Explanation or Formulae S.I.Unit
Formulae
Q Charge
70. Capacitance C
= = [M–1 L–2 T4 A2] F(Farad)
V Potential

dE Voltage × time
=L =
71. Self (or) Mutual Inductance  dI  Current [M1 L2 T–2 A–2] H (or) Wb/amp
 
 dt 

q .q
72. Electrical permittivity of free space ε0 = 1 2 2 [M–1 L–3 T4 A2] farad/m
4πFd

Charge
73. Surface charge density [M0 L–2 T1 A1] Cm–2
area

Light energy
74. Luminous flux [M1 L2 T–3] Lumen
time

∆E  Luminious flux  Lumen m–2 (or)


75. Intensity of illumination (or) Iluminance =I =   [M1 L0 T–3]
∆t.∆A  area  Lux.

1
76. Focal Power P= [M0 L–1 T0] Dioptre
focal length

1
77. Wave number (Propagation constant) v= [M0 L–1 T0] m–1
λ

Physical Quantities Having Same Dimensional S. Dimensional


Physical Quantities
Formulas No. Formulas
9. Angular velocity, Frequency, Velocity [T–1]
S. Dimensional
Physical Quantities gradient, Decay constant, rate of
No. Formulas
disintegration
1. Distance, Displacement, radius, light [L]
year wavelength, radius of gyration 10. Stress, Pressure, Modulus of Elasticity, [M L–1 T–2]
Energy density
2. Speed, Velocity, Velocity of light (c), [L T–1]
Relative velocity 11. Latent heat, Gravitational potential [L2 T–2]

3. Acceleration, acceleration due to [L T–2] 12. Specific heat, Specific gas constant [L2 T–1 q–1]
gravity, intensity of gravitational field, 13. Thermal capacity, Entropy, Boltzmann [M L2 T–2 q–1]
centripetal acceleration constant, Molar thermal capacity
4. Impulse, Momentum, Change in [M L T–1] 14. Wave number, Power of a lens, Rydberg [L–1]
momentum constant
5. Force, Weight, Tension, Thrust [M L T–2] 15. L [T]
Time, RC,, , LC
6. Work, Energy, Moment of force or [M L2 T–2] R
Torque, Moment of couple, heat Where L = Inductance, R = Resistance,
C = Capacitance
7. Force constant, Surface Tension, Spring [M T–2]
constant, Energy per unit area, Force 16. Power, Rate of dissipation of energy [M L2 T–3]
gradient 17. Intensity of sound, Intensity of radiation [M T–3]
8. Angular momentum, Angular impulse, [M L2 T–1] 18. Expansion coefficient, Temperature [K–1]
Planck’s constant coefficient of resistance

12 P NEET (XI) Module-1 PHYSICS


W
S. Dimensional When an object located against a specific point on a wall is
Physical Quantities
No. Formulas viewed with only one eye, say first with left eye and next with
19. Electric potential, Potential difference, [M L2 T–3 A–1] right eye, then the position of the object seems to change with
electromotive force respect to the point on the wall. This is called Parallax. The
20. Intensity of magnetic field, Intensity of [L–1 A] distance between the two points of observation is called the
magnetization Basis.
The distance D of a far away planet ‘P’ is measured by observing
21. Electric field and potential gradient [M L T–3 A–1]
it from two different places on earth say A and B separated by
22. Strain, Poisson’s ratio, refractive [M0 L0 T0] distance AB = b as shown in the figure.
index, dielectric constant, coefficient of (Dimensionless)
friction, relative permeability, Magnetic
susceptibility, Electric susceptibility,
angle, solid angle, Trigonometric ratios,
exponential constant
23. Solar constant and poynting vector [M L0 T–3]

LIMITATIONS OF DIMENSIONAL ANALYSIS The angle θ between the two directions along which the
METHOD planet is viewed at the two points is measured, which is called
parallax angle or parallactic angle. Since θ is small, AB can be
There are some limitations of Dimensional Analysis, which are considered as an arc of length of a circle with center at P and
given below: the distance D as radius
1. Dimensionless quantities cannot be determined by this
b
method. Constant of proportionality cannot be determined ∴ b= D θ ⇒ D=
by this method. They can be found either by experiment (or) θ
by theory. (e.g., universal gravitational constan G, Planck’s Once the distance ‘D’ of a planet is determined, the diameter
constant h, etc.) ‘d’ and angular size of planet can be estimated by same method.
2. This method is not applicable to trigonometric, logarthmic Two diametrically opposite points M & N of planet are viewed
and exponential functions. (e.g., y = A sin(wt – kx)) through telescope from a point A on earth. The angle a between
3. In the case of physical quantities which are dependent upon the two directions viewed is measured. Then by considering
more than three physical quantities, this method will be difficult. MN as arc of length of a circle with centre at A and the distance
4. In some cases, the constant of proportionality also possesses D as radius, we can write
dimensions. In such cases we cannot use this system.
d
5. If one side of equation contains addition or subtraction of d = Dα ( or ) α =
physical quantities, we cannot use this method. D
1 Accuracy and Precision
(e.g., s = ut + at 2 , v =
u + at ) The numerical values obtained on measuring physical
2
6. Same or may example dimensional quantities may not quantities depend upon the measuring instruments and methods
be unique. for work, energy and torque all have the same of measurement.
dimensional formula ML2T–2. A unit of measurement of a physical quantity is the standard
7. It gives no information whether a physical quantity is scalar reference of the same physical quantity which is used for
or vector. comparison of the given physical quantity.
Accuracy refers to how closely a measured value agrees with
Key Note the true values.
Precision refers to what limit or resolution the given physical
P A dimensionless quantity may have units, (e.g., radian, quantity can be measured
steradian) but a unit-less quantity has to be dimensionless. Accuracy refers to the closeness of observed values to its
true value of the quantity while precision refes to closeness
EXTENDED LEARNING between the different observed values of the same quantity.
High precision does not mean high accuracy. The difference
MEASUREMENT OF LARGE DISTANCES between accuracy and precision can be understand by the
Larger distances such as the distance of a planet or a star from following example: Suppose three students are asked to find
the earth cannot be measured directly with a metre scale. An the length of a rod whose length is known to be 2.250 cm. The
important method in such cases is the parallax method. observations are given in the table.

Units and Measurements 13


Putting improper values of the observations in calculation.
Student Measure- Measure- Measure- Average To minimise these error, sincerity and alertness of observer is
ment-1 ment-2 ment-3 length required.
A 2.25cm 2.27cm 2.26cm 2.26cm
Key Note
B 2.252cm 2.250cm 2.251cm 2.251cm
C 2.250cm 2.250cm 2.251cm 2.250cm P Personal errors like parallax error can be avoided by
It is clear from the above table , that the observation taken by a taking proper care.
student A are neither precise nor accurate. The observations of P The instrumental errors are avoided by calibrating the
student B are more precise. The observations of student C are instrument with a standard value and by applying proper
precise as will as accurate. corrections.

Error
Uncertainty in measurement of a physical quantity is called the TRUE VALUE AND VARIOUS
error in measurement. REPRESENTATION OF ERRORS
OR
The difference between true value and measured value (as per the True Value
standard) is called as error.
In the measurement of a physical quantity the arithmetic mean of
Types of Errors all readings which is found to be very close to the most accurate
Based on the source they arise form, there are three types of error. reading is to be taken as True value of the quantities. If a1, a2, a3
1. Systematic Errors: The errors due to a definite cause and 1 n
which follow a particular rule are called systematic errors. ............ an are readings then true value amean = n ∑ ai
i =1
As we know the cause of these errors, so can be minimized.
They always occur in one direction. Following are some
Absolute Errors
systematic errors. The magnitude of the difference between the true value of
(i) Instrumental Error: These error are due to imperfect the measured physical quantity and the value of individual
design of measurement instrument. These can be measurement is called absolute error.
minimize of by using more accurate instrument. Absolute Error = |True value-measured values|
(ii) Environmental Error: The error arised due to change Dai = |amean – ai|
in external environmental conditions like changes in
temperature, pressure, humidity etc. The absolute error is always positive and has unit.
e.g., Due to rise in temperature, a scale gets expanded Mean Absolute Error
and this results in error in measuring length. The arithmetic mean of all the absolute errors is considered as
(iii) Imperfection in Experimental Technique or Procedure: the mean absolute error or final absolute error of the value of the
The error due to experimental arrangement, procedure physical concerned.
followed and experimental technique is called ∆a1 + ∆a2 + − − − ∆an 1 ∞
Imperfection error. ∆amean =
n
= ∑ ∆ai
n i =1
e.g., In calorimetric experiments, the loss of heat due to
radiation, cannot be avoided. The mean absolute error is always positive.
(iv) Personal Errors or Observational Errors: These errors Relative Error
are entirely due to personal peculiarities like individual
The relative error of a measured physical quantity is the ratio of the
bias, lack of proper settings of the apparatus carelessness
mean absolute error to the mean value of the quantity measured.
in taking observations.
1 ∆amean
Probable error ∝ Relative error =
no. of readings amean
e.g., Parallax error. It is a pure number having no units.
2. Random Errors: They are due to uncontrolled disturbances
and unknown causes which influence the physical quantity
Percentage Error
and the instrument.  ∆a 
=δa  mean × 100  %
Due to unknown causes they are irregular and variable in  amean 
magnitude and sign. They are not eliminated completely.
Random error can be minimized by repeating the observations
Key Note
a large number of times and taking the arithmetic mean of all.
3. Gross Errors: These arise due to observer carelessness and Relative error and percentage error give a measure of accuracy
mistake during reading and calculating the measured result. i.e., percentage error increases accuracy decreases.
e.g., Reading instrument without proper initial settings.

14 P NEET (XI) Module-1 PHYSICS


W
Combination of Errors As we check for maximum error a + ve sign is to be taken for
Error due to addition ∆C
™
the term r
If Z = A + B C
DZ = DA + DB (Max. Possible absolute error) Maximum Percentage error in Z
Z ± DZ = (A ± DA) + (B ± DB) ∆Z ∆A ∆B ∆C
× 100= p × 100 + q × 100 + r × 100
Z A B C
∆Z ∆A + ∆B
Relative error
= =
Z A+ B
∆Z ∆A + ∆B
Percentage error = × 100 = × 100% Train Your Brain
Z A+ B
™ Error due to subtraction
If Z = A – B Example 9: In an experiment, two resistances measured
DZ = DA + DB (Max. Possible absolute error) are (1.3 ± 0.1)W and (2.4 ± 0.2) W. Calculate the equivalent
resistance if the two resistances are connected in series
Z ± DZ = (A ± DA) – (B ± DB)
with percentage error
∆Z ∆A + ∆B Sol. Here, R1 = (1.3 ± 0.1)W and R2 = (2.4 ± 0.2)W.
Relative error
= =
Z A− B In parallel, Rs = R1 + R2 = 1.3 + 2.4 = 3.7 W
∆Z ∆A + ∆B DRs = ± (DR1 + DR2) = ± (0.1 + 0.2) = ± 0.3
Percentage error = × 100 = × 100%
Z A− B 0.3
Whether it is addition or subtraction, maximum absolute error % error = ± × 100 =±8.1%
™ 3.7
is same.
Hence, Rs = (3.7 ± 0.3)W = 3.7W ± 8.1%
™ In subtraction the percentage error increases.
™ Error due to Multiplication
∆Z ∆A ∆B
If Z = AB then = +
Z A B
∆Z Concept Application
is called fractional error or relative error.
Z
∆Z 12. If L = 2.06 cm ± 0.02 cm,
Percentage error = × 100
Z B = 1.11 cm ± 0.03 cm, then L + B equals to
 ∆A   ∆B  (1) 3.17 cm ± 0.05 cm (2) 2.06 cm ± 0.05 cm
= × 100  +  × 100 
 A   B  (3) 3.17 cm ± 0.02 cm (4) 3.17 cm ± 0.03 cm
13. The radius of sphere is measured as (5.2 ± 0.2) cm then
Here Percentage error is the sum of individual percentage
errors. the percentage error in volume of the ball is
™ Error due to Division:
(1) 11% (2) 4 % (3) 7% (4) 9%
14. If the length and breadth of a plate are (5.02 ± 0.2) cm and
A (4.0 ± 0.1) cm then the absolute error in measurement of
Z=
B area is
Maximum possible relative error (1) 10 cm2 (2) 11 cm2
∆Z ∆A ∆B (3) 12 cm 2
(4) 1.3 cm2
= + 15. If the length of a cylinder is measured to be 8.28 cm with
Z A B
an error of 0.01 cm then the percentage error in measured
Max. Percentage error in division length is nearly
∆Z ∆A ∆B (1) 0.4% (2) 0.2% (3) 0.1% (4) 0.5%
× 100 = × 100 + × 100
Z A B 16. A body travels uniformly a distance of (13.8 ± 0.2) m
™ Error due to Power in a time (4.0 ± 0.3) s then the velocity of the body is
If Z = An (1) (3.45 ± 0.2) ms–1 (2) (3.45 ± 0.3) ms–1
∆Z ∆A (3) (3.45 ± 0.4) ms –1
(4) (3.45 ± 0.5) ms–1
=n 17. The pressure on a square plate is measured by measuring
Z A
the force on the plate and the length of the sides of the
™ In more general form
plate. If the maximum error in measurement of force and
A p Bq length are respectively 4% and 2% then the maximum
If Z = then maximum fractional error in Z is
Cr error in Measurement of pressure is:
∆Z ∆A ∆B ∆C (1) 1% (2) 2%
= p +q +r (3) 6% (4) 8%
Z A B C

Units and Measurements 15


Order of Magnitude Rule 5: I n exponential notation, the numerical portion gives the
The order of Magnitude of a physical quantity is the power of number of significant figures.
10 which is required to represent the physical quantity. It gives e.g., 1.32 × 10–2 has three significant figures.
an idea about how big or how small a given physical quantity is: 1.32 × 104 has three significant figures.
To determine the order of magnitude of a number Z, we first Rule 6: T he trailing zeros in a number without a decimal point
express it as will not be significant.
Z = Y × 10x e.g., 500 – has one significant figures
If 0.5 < Y ≤ 5 then x will be the order of magnitude of Z. 101 has three significant figures.
Rule 7:When the number is expressed in exponential form, the
Some Examples are given below:
exponential term does not affect the number of S.F. For
Measure Expressed Nearest Order of example in x = 12.3 = 1.23 × 101 = .123 × 102 = 0.0123 ×
No. Z power of 10 Magnitude 103 = 123 × 10–1, each term has 3 SF only.

7 0.7 × 101 1 S. No. Number Significant figures


48 4.8 × 101 1 1. 2.003 4
51 0.51 × 102 2 2. 0.002 1
554 0.554 × 10 3
3 3. 0.0020 2
997 0.997 × 10 3
3 4. 2.4001 5
1003 1.003 × 103 3 5. 1000 1
756000 0.756 × 106 6 6. 1000 kg 4
0.136 1.36 × 10 –1
–1 7. 1.0 × 10 6
2
0.05 5 × 10 –2
–2 8. 1.00 × 10 6
3
0.92 0.92 × 100 0
Rounding off Numbers
Significant Figures The process of omitting the non significant digits and retaining
only the desired number of significant digits, incorporating
A significant figure is defined as the figure, which is considered
reasonably, trust worthy in number. the required modifications to the last significant digit is called
rounding off the number.
e.g., p = 3.141592654 (upto 10 digits)
= 3.14 (with 3 figures) Rules for Rounding off Numbers
= 3.1416 (upto 5 digits) Rule 1: If the digit to be dropped is less than 5, then the preceding
Larger the number of significant figures obtained in the digit is left unchanged.
measurement, grater will be the accuracy. e.g., (i) x = 7.82 is rounded off upto two S. F. 7.8
e.g., (ii) x = 3.94 is rounded off upto two S. F. 3.9.
Rules for Determining the Number of Significant Rule 2: If the digit to be dropped is more than 5, then the
Figures preceding digit is raised by 1.
Rule 1: All Non-zero digit in a given no. are significant without e.g., (i) x = 6.87 is rounded off upto two S.F. 6.9.
any regard to its place.
e.g., (ii) x = 12.78 is rounded off upto three S.F. 12.8.
e.g., 42.3 has three significant figures.
Rule 3: If the digit to be dropped is 5 followed by digits other
243.4 has four significant figures. than zero, then the preceding digit is raised by 1.
Rule 2: A zero becomes a significant figure if it appears between e.g., (i) x = 16.351 is rounded off upto three S.F. 16.4.
two non-zero digits.
e.g., (ii) x = 6.758 is rounded off upto two S.F. 6.8.
e.g., 5.03 has three significant figures.
Rule 4: If the digit to be dropped is 5 or 5 followed by zeros, then
5.604 has four significant figures. the preceding digit is left unchanged, if it is even.
Rule 3: Leading zeros or the zeros placed to the left of the number e.g., (i) x = 3.250 becomes 3.2 on rounding off upto two
are never significant. S.F.
e.g., 0.543 has three significant figures. e.g., (ii) x = 12.650 becomes 12.6 on rounding off upto
0.006 has one significant figure. three S.F.
Rule 4: Trailing zeros or the zeros placed to the right of the Rule 5: If the digit to be dropped is 5 or 5 followed by zeros, then
number with decimal point are significant. the preceding digit is raised by 1, if it is odd.
e.g., 4.330 has four significant figures. e.g., (i) x = 3.750 is rounded off upto two S.F. 3.8.
433.00 has five significant figures. e.g., (ii) x = 16.150 is rounded off upto three S.F. 16.2.

16 P NEET (XI) Module-1 PHYSICS


W
Rules for Arithmetic Operations with Significant
Example 11: Each side of a cube is measured to be 7.203 m.
Figures
Find the volume of the cube up to appropriate significant
1. In addition and subtraction the number of decimal places in figures.
the result should be equal to the least decimal place that the Sol. Volume = a3 = (7.023)3 = 373.715 m3
given number have. The rule is illustrated by the following Answer must have 4 SF.
examples: So, correct answer is 373.7
Example 12: The mass of a box is 2.3 kg. Two marbles of
(a) 33.3 (has only one decimal place) masses 2.15 g and 12.39 g are added to it. Find the total
3.11 mass of the box to the correct number of significant figures.
+ 0.313
Sol. Total mass = 2.3 + 0.00215 + 0.01239 = 2.31 kg
36.723 (answer should be rounded off one
decimal place) The total mass in appropriate significant figures will be
2.3 kg.
Answer = 36.7
(b) 3.1421
0.241
+ 0.09 (has 2 decimal place)
  3.4731 (answer should be rounded off 2 decimal Concept Application
places)
Answer = 3.47 18. 2.34 is obtained by rounding off the number
(c) 62.831 (has 3 decimal place) (1) 2.346 (2) 2.355
– 24.5492 (3) 2.335 (4) 2.334
  38.2818 (answer should be rounded off 3 decimal
19. The number of significant figures in 0.0006032 is
places)
(1) 7 (2) 4
Answer = 38.282
(3) 3 (4) 5
2. In multiplication or division the number of significant
20. The radius of disc is 1.2 cm, its area according to idea
figure in result is equal to the smallest significant figure of significant figures is _______ .
that the given number have. The rule is illustrated by the (1) 4.5216 cm2 (2) 4.521 cm2
following examples:
(3) 4.52 cm 2
(4) 4.5 cm2
(a) 142.06
21. When Energy is expressed in erg the no. of significant
× 0.23 (two significant figures)
32.6738 (answer should have two significant figure is four. If it is expressed in joule the no. of
figures) significant figures will become
(1) 9 (2) 5
Answer = 33
(3) 1 (4) 4
(b) 51.028
× 1.31 (three significant figures)
66.84668
Answer = 66.8 VERNIER CALIPERS
(c) 0.90 It is a device used for accurate measurement. There are two scales
= 0.2112676
4.26 in the vernier calipers, vernier scale and main scale. The main
scale is fixed whereas the vernier scale is movable along the
Answer = 0.21 main scale.
Its main parts are as follows:
Train Your Brain 1. Main scale: It consists of a steel metallic strip M, graduated
in cm and mm at one edge and in inches and tenth of an inch
at the other edge on same side. It carries fixed jaws A and C
Example 10: Two bodies of masses (15.6 ± 0.2) gram and projected at right angle to the scale as shown in figure.
(23.9 ± 0.3) gram are put in a bag of negligible mass. What
D S
is the total mass of bag? C Main Scale
10 E
3
Sol. Sum of the masses = 15.6 + 23.9 = 39.5g 0 1 5 6 7 8 9
V M
Max. Error in sum of masses = ± (0.2 + 0.3) = ± 0.5g
\ Total mass = (39.5 ± 0.5) gram OP
A B

Units and Measurements 17


2. Vernier Scale: A vernier V slides on the strip M. It can be One of the following situations will arise.
fixed in any position by screw S. It is graduated on both 1. Zero of Vernier scale coincides with zero of main scale (see
sides. The side of the vernier scale which slides over the figure)
mm side has ten divisions over a length of 9 mm, i.e., over 9
main scale divisions and the side of the vernier scale which
0 0.5 M 1
slides over the inches side has 10 divisions over a length of
0.9 inch, i.e., over 9 main scale divisions.
3. Movable Jaws: The vernier scale carries jaws B and D
projecting at right angle to the main scale. These are called 0 5 V 10
movable jaws. When vernier scale is pushed towards A
In this case, zero error and zero correction, both are nil.
and C, then as B touches A, straight side of D will touch
straight side of C. In this position, an instrument is free Actual length = observed (measured) length.
from errors, zeros of vernier scale will coincide with zeros 2. Zero of vernier scale lies on the right of zero of main scale
of main scales, on both the cm and inch scales. this called as positive zero error.
(The object whose length or external diameter is to be
measured is held between the jaws A and B, while the 0 0.5 M 1
straight edges of C and D are used for measuring the internal
diameter of a hollow object).
4. Metallic Strip: There is a thin metallic strip E attached
to the back side of M and connected with vernier scale. 0 5 V 10
When the jaws A and B touch each other, the edge of Fig. 1:
strip E touches the edge of M. When the jaws A and B
3. Zero of the vernier scale lies left of the main scale this
are separated, E moves outwards. The strip E is used for
called as negative zero error.
measuring the depth of a vessel.
Least Count (Vernier Constant) of Vernier Caliper 0 0.5 M 1
The smallest value of the measurement that can be directly taken
from a measuring instrument is called its least count.
For Vernier Caliper: It is the difference between one main scale
division and one vernier scale division. 0 5 V 10
V.C./Least Count = 1 M.S.D – 1 V.S.D Fig. 2:
When there are n division on vernier scale coincides with (n –1)
division on the main scale then Experiment
n V.S.D = (n – 1) M.S.D Aim: To measure the diameter of a small spherical/cylindrical
n –1  body, using a vernier caliper.
1 V.S.D =   M.S.D
 n  Apparatus: Vernier caliper, a spherical (pendulum bob) or a
cylinder.
So, least count = 1 MSD – 1 V.S.D
S

 n –1   n − 1 C D Main Scale
= 1 M.S.D. –   M.S.D = 1 −  M.S.D 3
 n  n  E
 0 1
V
5 6 7 8 9 10
M

M.S.D
L.C. = P

n
In standard vernier 10 division of V.S. is coincide with 9 division A B
of M.S and 1 division of Main scale is equal to 0.1 mm. SPHERE

So, lease count = 1 M.S.D – 1 V.S.D Theory: If the body kept between the jaws, the zero of vernier
and 9 M.S.D = 10 V.S.D scale lies ahead of Nth division of main scale, then main scale
9 reading (M.S.R.) = N.
1V.S.D = M.S.D
10 If nth division of vernier scale coincides with any division of main
9 1 1 scale, then vernier scale reading (V.S.R.)
\ LC = 1M.S.D – M.S.D = M.S.D = (1mm) = 0.1mm
10 10 10 = n × (L.C.) (L.C. is least count of vernier caliper)
LC = 0.01 cm Total reading, T.R. = M.S.R. + V.S.R. = N + n × (V.C.)
Determination of Zero error and Zero Correction Precautions (to be taken)
For this purpose, movable jaw B is brought in contact with fixed 1. Motion of vernier scale on main scale should be made smooth
jaw A. (by oiling if necessary).

18 P NEET (XI) Module-1 PHYSICS


W
2. Vernier constant and zero error should be carefully found and
properly recorded.
3. The body should be gripped between the jaws firmly but Concept Application
gently (without undue pressure on it from the jaws).
4. Observations should be taken at right angles at one place and 22. A Vernier Callipers has 1 mm marks on the main scale. It
taken at least at three different places. has 20 equal divisions on the vernier scale, which match
Sources of Error with 16 main scale divisions. For this Vernier callipers,
the least count is
1. The vernier scale may be loose on main scale.
(1) 0.02 mm (2) 0.05 mm
2. The jaws may not be at right angles to the main scale.
3. The graduations on scale may not be correct and clear. (3) 0.1 mm (4) 0.2 mm
4. Parallax may be there in taking observations. 23. In an experiment, the angles are required to be measured
using an instrument, 29 divisions of the main scale
Calculation of Zero error for Vernier Calipers coincide with 30 divisions of the vernier scale. If the
Positive zero error = Number of division of V.S. coinciding × L.C. smallest division of the main scale is hald a degree (=
Negative zero error = (Total number of V.S. – Number of division 0.5°), then the least count of the instrument is
coinciding) L.C. (1) Half minute (2) One degree
(3) Half degree (4) One minute
Correct reading with Zero error
24. A spectrometer gives the following reading when used
Correct Reading = Reading to M.S – (Positive zero error)
to measure the angle of a prism.
Correct Reading = Reading to M.S + (Negative zero error)
Main scale reading : 58.5 degree
Vernier scale reading : 9 divisions.
Train Your Brain Given that 1 division on main scale corresponds to
0.5 degree. Total divisions on the vernier scale is 30 and
match with 29 divisions of the main scale. The angle of
Example 13: The least count of vernier caliper is 0.1 mm. the prism from the above data:
The main scale reading before the zero of the vernier scale (1) 58.59° (2) 58.77°
is 10 and the zeroth division of the vernier scale coincides
(3) 58.65° (4) 59°
with the main scale division. Given that each main scale
division is 1 mm, what is the measured value?
Sol. Length measured with vernier caliper = reading before
the zero of vernier scale + number of vernier divisions SCREW GAUGE
coinciding with any main scale division × least count
= 10 mm + 0 × 0.1 mm = 10 mm = 1.00 cm This instrument (shown in figure) works on the principle of
Example 14: A vernier caliper has its main scale of 10 cm micrometer screw. It consists of a U-shaped frame M. At one end
equally divided into 200 equal parts. Its vernier scale of 25 of it is fixed a small metal piece A of gun metal. It is called stud
divisions coincides with 24 division on the main scale. Find and it has a plane face. The other end N of M carries a cylindrical
the least count of the instrument. hub H.
Sol. In vernier caliper main scale 10 cm.
The hub extends few millimetre beyond the end of the frame. On
10
10 cm divided in 200 divisions; 1 MSD = = 0.05 cm. the tubular hub along its axis, a line is drawn known as reference
200
line. On the reference line graduations are in millimetre and half
here, 25 VSD = 24 MSD.
millimeter depending upon the pitch of the screw.
24
1 VSD = MSD This scale is called linear scale or pitch scale. A nut is threaded
25
Least count = 1 MSD – 1 VSD through the hub and the frame N through the nut screw S can move
24 up to stud A. The front face B of the screw, facing the plane face
Least count = 1 MSD – MSD A, is also plane. A hollow cylindrical cap K, is capable of rotating
25
over the hub when screw is rotated.
1
Least count = MSD It is attached to the right hand end of the screw. As the cap is
25
1 rotated the screw either moves in or out. The bevelled surface E
= ×0.05 cm of the cap K is divided into 50 or 100 equal parts. It is called the
25
Least count = 0.002 cm circular scale or head scale. Right hand end R of K is milled for
proper grip.

Units and Measurements 19


N H 2. Zero mark of circular scale remains on right of reference
AB S
E K line and does not cross it (see figure).
0
Stud
Sleeve Ratchet R Circular
Scale
5 0

Frame
Reference
M H line
In most of the instrument the milled head R is not fixed to the
screw head but turns it by a spring and ratchet arrangement such
that when the body is just held between faces A and B, the spring N
yields and milled head R turns without moving in the screw.
In an accurately adjusted instrument when the faces A and B are
just touching each other, the zero marks of circular scale and pitch Here 2nd division on circular scale comes on reference line.
scale exactly coincide. Zero reading is already 0.02 mm. It makes zero error + 0.02
Determination of Least Count of Screw Gauge mm and zero correction – 0.02 mm.
Note the value of linear (pitch) scale division. Rotate screw to bring Actual thickness will be 0.02 mm less than the observed
zero mark on circular (head) scale on reference line. Note linear (measured) thickness.
scale reading i.e. number of divisions of linear scale uncovered by 3. Zero mark of circular scale goes to left on reference line
the cap. after crossing it (see figure). Here zero of circular scale
Now give the screw a few known number of rotations. (one
has advanced from reference line by 3 divisions on circular
rotation completed when zero of circular scale again arrives on the
reference line). Again note the linear scale reading. Find difference scale. A backward rotation by 0.03 mm will make reading
of two readings on linear scale to find distance moved by the screw. zero. It makes zero error – 0.03 mm & zero correction +
Distance moved by in n rotation 0.03 mm.
Then, pitch of the screw =
No. of full rotation (n)
Now count the total number of divisions on circular (head) scale. Circular
Pitch
Scale
Then, least count = 0 3 5
Total number of divisions on the circular scale
The least count is generally 0.001 cm. Reference
Determination of Zero Error and Zero Correction H line
For this purpose, the screw is rotated forward till plane face B of
the screw just touches the fixed plane face A of the stud and edge
of cap comes on zero mark of linear scale. Screw gauge is held N
keeping the linear scale vertical with its zero downwards.
One of the following three situations will arise. Actual thickness will be 0.03 mm more than the observed
1. Zero mark of circular scale comes on the reference line (measured) thickness.
(see figure)
Experiment
Circular Aim: To measure diameter of a given wire using a screw gauge
Scale and find its volume.
0 O 95
Apparatus: Screw gauge, wire, half metre rod (scale).
Reference Theory
H line
1. Determine of least count of screw gauge
2. If with the wire between plane faces A and B, the edge of
N
the cap lies ahead of Nth division of linear scale.

In this case, zero error and zero correction, both are nil Then, linear scale reading (L.S.R.) = N
Actual thickness = Observed (measured) thickness. If nth division of circular scale lies over reference line.

20 P NEET (XI) Module-1 PHYSICS


W
Then, circular scale reading (C.S.R.) = n × (L.C.)
(L.C. is least count of screw gauge) Sol. Pitch = 0.5 mm, L.C. = 0.001 cm
Total reading (T.R.) = L.S.R. + C.S.R. = N + n × (L.C.) Pitch of screw = Linear distance traveled in one Revolution
3. If D be the mean diameter and l be the mean length of the 2mm
=P = 0.5=
mm 0.05 cm
D
2
4
wire. Then volume of the wire, V = π   l Pitch
2 Least count =
wire no. of divisions in circular scale
S N H 0.05
A B E = = 0.001 cm
K 50
0
Example 16: The pitch of a screw gauge 0.5 mm and
R there are 50 divisions on the circular scale. In measuring
the thickness of a metal plate, there are five divisions on
the pitch scale (or main scale) and thirty fourth division
coincides with the reference line. Calculate the thickness
of the metal plate.
M Sol. Main Scale reading = 5 × 0.5 = 2.5 mm
Calculation 0.5 mm
Least count = = 0.01 mm
Mean diameter of the wire, 50 div.
D1 + D2 + ..... + D9 + D10  Thickness = Main Scale + (Coinciding division of
D = ...... mm = … cm
10 circular scale × L.C.)
Mean length of the wire, = 2.5 mm + (34 × 0.01) mm
= 2.5 mm + 0.34 mm
l1 + l2 + l3
=l = ....cm = 2.84 mm
3
Volume of the wire
2
D
V = π   l = ...cm3
2 Concept Application
Result The volume of the given wire is = … cm 3

Precaution (to be taken) 25. Two full turns of the circular scale of a screw gauge
cover a distance of 1 mm on its main scale. The total
1. While taking an observation, the screw must always be
number of divisions on the circular scale is 50. Further, it
turned only in one direction so as to avoid the backlash
error. is found that the screw gauge has a zero error of -0.03 mm
while measuring the diameter of a thin wire, a student
2. At each place, take readings in pairs i.e. in two directions at
notes the main scale reading of 3 mm and the number
right angles to each other.
of circular scale divisions in line with the main scale as
3. The wire must be straight and free from kinks.
35. The diameter of the wire is:
4. Always rotate the screw by the ratchet and stop as soon as
(1) 3.32 mm (2) 3.37 mm
it gives one tick sound only.
(3) 3.67 mm (4) 3.38 mm
Sources of Error 26. 10 rotations of the cap of a screw gauge is equivalent to
1. The screw may have friction. 5 mm. The cap has 100 divisions. Find the least count.
2. The screw gauge may have back-lash error. a reading taken for the diameter of wire with the screw
3. Circular scale divisions may not be of equal size. gauge shows four complete rotations and 35 divisions
4. The wire may not be uniform. on the circular scale. Find the diameter of the wire.
27. A screw gauge gives the following reading when used
Train Your Brain to measure the diameter of a wire.
Main scale reading: 0 mm
Circular scale reading: 52 divisions
Example 15: In four complete revolutions of the cap, the
Given that 1 mm on the main scale corresponds to 100
distance traveled on the pitch scale is 2mm. If there are fifty
divisions of the circular scale
divisions on the circular scale, then
The diameter of the wire from the above date is
(a) Calculate the pitch of the screw gauge
(1) 0.026 cm (2) 0.005 cm
(b) Calculate the least count of the screw gauge (3) 0.52 cm (4) 0.052 cm

Units and Measurements 21


SHORT NOTES
Fundamental Quantity Derived Quantity Significant Figure or Digits
The physical quantities which Those quantities which can Rules to find out the number of significant figures:
do not depend on any other be expressed in terms of 1. Rule: All the non-zero digits are significant e.g., 1984 has
physical quantities for their fundamental/base quantities. 4 SF.
measurements. e.g., Angle, speed or velocity 2. Rule: All the zeros between two non-zero digits are significant.
e.g., Mass, Length, Time Acceleration, force etc. e.g., 10806 has 5 SF.
Temperature, current, luminous 3. Rule: All the zeros to the left of first non-zero digit are not
Intensity & mole significant. e.g., 00108 has 3 SF.
4. Rule: If the number is less than 1, zeros on the right of the
System of Units decimal point but to the left of the first non-zero digit are not
(a) FPS System: Here length is measured in foot, mass in significant. e.g., 0.002308 has 4 SF.
pounds and time in second. 5. Rule: The trailing zeros (zeros to the right of the last non-zero
digit) in a number with a decimal point are significant. e.g.,
(b) CGS System: In this system, L is measured in cm, M is
01.080 has 4 SF.
measured in g and T is measured in sec.
6. Rule: The trailing zeros in a number without a decimal point
(c) MKS System: In this system, L is measured in metre, M is are not significant e.g., 010100 has 3 SF. But if the number
measured in kg and T is measured in sec. comes from some actual measurement then the trailing zeros
become significant. e.g., m = 100 kg has 3 SF.
Principle of Homogeneity
7. Rule: When the number is expressed in exponential form,
According to this, the physical quantities having same dimension the exponential term does not affect the number of S.F. For
can be added or subtracted with each other and for a given equation, example in x = 12.3 = 1.23 × 101 = .123 × 102 = 0.0123 × 103
dimensions of both sides must be same. = 123 × 10–1, each term has 3 SF only.
B Rules for arithmetical operations with significant figures:
For eg, in equation F
= A m+ +C ,
v 1. Rule: In addition or subtraction the number of decimal places
all the three parts of R.H.S have same dimension as force on L.H.S. in the result should be equal to the number of decimal places
of that term in the operation which contain lesser number of
Dimensions decimal places. e.g., 12.587 – 12.5 = 0.087 = 0.1 ( second
term contain lesser i.e., one decimal place)
The fundamental or base quantities along with their powers needed
to express a physical quantity is called dimensions 2. Rule: In multiplication or division, the number of SF in the
product or quotient is same as the smallest number of SF in
e.g., [F] = [MLT–2] is dimension of force.
any of the factors. e.g., 5.0 × 0.125 = 0.625 = 0.62.
Usage of Dimensional Analysis
Rounding Off
(i) To check the correctness of a given formula.
Rules for rounding off the numbers:
(ii) To establish relation between quantities dimensionally.
1. Rule: If the digit to be rounded off is more than 5, then the
(iii) To convert the value of a quantity from one system of
preceding digit is increased by one. e.g., 6.87≈ 6.9
units to other system.
2. Rule: If the digit to be rounded off is less than 5, than the
Limitations of Dimensional Analysis preceding digit is unaffected and is left unchanged. e.g., 3.94
(i) It does not predict the numerical value or number ≈ 3.9
associated with a physical quantity in a relation 3. Rule: If the digit to be rounded off is 5 then the preceding
digit is increased by one if it is odd and is left unchanged if it
e.g., v= u + 1 at & v = u + at is even. e.g., 14.35 ≈ 14.4 and 14.45 ≈ 14.4
3 5
Both are dimensionally valid. Representation of Errors
(ii) It does not derive any relations involving trigonometric, 1. Mean absolute error is defined as
logarithmic and exponential functions ∆a1 + ∆a2 + ... + ∆an n
∆a
e.g., P = P0e–bt cannot be derived dimensionally.=
2 ∆a = ∑ i
n i =1 n
(iii) It does not give any information about dimensionally
Final result of measurement may be written as:
constants or nature of a quantity (vector/scalar) associated
with a relation. a = am ± ∆a

22 P NEET (XI) Module-1 PHYSICS


W
2. Relative Error or Fractional Error: It is given by To Find Smaller Measurements
∆a Mean absolute Error Vernier Calliper
=
a
m
Mean value of measurement (i) Least count: Suppose movable Jaw is slided till the zero
of vernier scale coincides with any of the mark of the main
∆a scale.
3. Percentage Error
= × 100%
am  n −1 
Let, n V.S.D = (n – 1) MSD ⇒ 1VSD =   M.S.D
 n 
Combination of Errors
\ Vernier constant = 1 M.S.D – 1 V.S.D
(i) In Sum: If Z = A + B, then ∆Z = ∆A + ∆B.
 n − 1 1
Maximum fractional error in this case is
= 1 − n  MSD = n
MSD
 
∆Z ∆A ∆B

= + (ii) Total reading = MSR + VSR
Z A+ B A+ B
= MSR + n ×VC
(ii) In Difference: If Z = A – B, then maximum absolute error where MSR = Main scale reading
is ∆Z = ∆A + ∆B and maximum fractional error in this case VC = Vernier constant i.e. least count
∆Z ∆A ∆B n = nth division of vernier scale coinciding with main scale.
= +
Z A− B A− B Screw Gauge
(iii) In Product: If Z = AB, then the maximum fractional error, This instrument works on the principle of micro-meter screw. It
is used to measure very small (mm) measurements. It is provided
∆Z ∆A ∆B
= + with linear scale and a circular scale.
Z A B
(i) Pitch of the screw gauge
(iv) In Division: If Z = A/B, then maximum fractional error is Distance moved in n-rotation of cir-scale
=
∆Z ∆A ∆B No.of full -rotation
= +
Z A B
Pitch
∆Z ∆A (ii) L.C =
(v) In Power: If Z = An then =n Total number of division on the circular scale
Z A (iii) Total Reading (T.R) = L.S.R + C.S.R
Ax B y L.S.R = Linear scale Reading = N where
In more general form if Z =
Cq C.S.R = Circular Scale Reading = n × L.C
then the maximum fractional error in Z is If nth division of circular scale coincides with the linear scale
∆Z ∆A ∆B ∆C line, then
=x +y +q \ Total reading = N + n × (L.C)
Z A B C

Units and Measurements 23


AARAMBH (SOLVED EXAMPLES)
1. A new system of units is proposed in which, unit of mass is Sol.
c = [ M0 L1 T–1], G = [ M–1 L3 T–2], h = [ M1 L2 T–1]
α kg, unit of length is β m and unit of time is γ s. What will T = ca Gb hc
be value of 5 J in this new system? [T1] = [ M0 L1 T–1]a [ M–1 L3 T–2]b [ M1 L2 T–1]c
(1) 5αβ2γ–2 (2) 5α–1β–2γ2
[T1] = M –b + c La + 3 b + 2c T– a – 2b – c
(3) 5α β γ
–2 –1 –2
(4) 5α–1β2γ2
On comparing,
Sol. Joule is a unit of energy. ∴– b + c = 0, a + 3b + 2c = 0 and a + 2b + c = –1
n1 = 5 n2 = ? Solving the three equations
M1 = 1 kg M2 = α kg a = –5/2, b = 1/2, c = 1/2
L1 = 1 m L2 = β m T = [c–5/2 G1/2 h1/2]
T1 = 1 s T2 = γ s Therefore, option (1) is the correct answer.
Dimensional formula of energy is [ML2T–2]. Comparing with 5. By what percentage should the pressure of a given mass of
[MaLbTc], we get gas be increased so as the decrease in its volume is 10% at
a = 1, b = 2, c = –2 a constant temperature?
a b c (1) 5 % (2) 7.2 %
M  L  T 
As, n 2 = n1  1   1   1  (3) 12.5 % (4) 11.1 %
 M 2   L 2   T2  Sol. PV = constant
1 2 −2
 1kg   1m   1s  5γ 2  10V 
= 5      = 2
= 5α −1β−2 γ 2 P ' V − = PV
 α kg   β m γ
   s αβ  100 
Therefore, option (2) is the correct answer.  100V − 10V 
P '  = PV
 100 
 2π 
2. Given
= that y A sin  ( ct − x )   where y and x are  90 V  10
 λ  P '   PV  P '  P
measured in meter. Which of the following statements are true?  100  9
(1) The unit of λ is same as that of x and A Percentage increase in pressure
(2) The unit of λ is same as of x but not of A  P1 − P 

=   × 100%
(3) The unit of c is same as that of  P 
λ  10 
2π =  p − p  × 100%
(4) The unit of (ct – x) is same as that of
λ  9 
 2π   P = 11.1 %
Sol. f  A sin    ct  x . Since Angle has [MoLoTo] Therefore, option (4) is the correct answer.
 λ  
6. The percentage error in the measurement of mass and speed
 2π 
dimensions.   ct  x  needs to be dimensionless
are 2% and 3% respectively. The error in the estimation of
λ  kinetic energy obtained by measuring mass and speed will be:
(1) 12% (2) 10%
x = λ =  M 0 L1T 0  , A = [M0 L1 T0] (3) 2% (4) 8%
1
Therefore, option (1) is the correct answer. Sol. K = mv 2
2
3. If the unit of force and length are doubled, the unit of energy ∆K ∆m 2∆v
× 100 = × 100 + × 100
will be: K m v
(1) 1/4 times the original (2) 1/2 times the original ∆K
(3) 2 times the original (4) 4 times the original × 100= 2% + 2 × 3%= 8%
K
Sol. Work = Energy = 2 F × 2 L = 4 times the original Therefore, option (4) is the correct answer.
Therefore, option (4) is the correct answer.
7. If E, m, l and G denote energy, mass, angular momentum and
4. The speed of light c, gravitational constant G and Planck’s  El2 
constant h are taken as fundamental units in a system. gravitational constant respectively, the quantity  5 2 
The dimensions of time in this new system should be: has the dimensions of: m G 
(1) [G1/2h1/2c–5/2] (2) [G–1/2h1/2c–1/2] (1) Mass (2) Length
(3) [G–1/2 h1/2c–3/2] (4) [G–1/2 h1/2c1/2] (3) Time (4) Angle

24 P NEET (XI) Module-1 PHYSICS


W
 El 2  [ M 1 L2T −2 ][ M L2 T −1 ]2 Sol. Displacement in tth second = Displacement in t sec –
Sol.  5 2 = Displacement in (t – 1) sec
m G  [ M 5 ][ M −2 L6T −4 ]
 1   1 
 M 3L6T 4  =  ut  at 2   u (t  1  a (t  1) 2 
  3 6 4    M 0 L0T 0  = Dimensionless
 2   2 
 M L T  1 2 1
= ut  at  ut  u (1)  a t 2  2t (1)  (1) 2 
Therefore, option (4) is the correct answer. 2 2
1 1 a
A4B
1
3 = ut  at 2  ut  u (1)  at 2  at (1)  (1) 2
8. If Z = and ∆A, ∆B, ∆C and ∆D are their absolute
43 2 3
1
2 2 2
CDAB a 2
Z = u (1)  at (1) 
errors in A,CD
B,3C
2 and D respectively. The relative error in Z is:
(1)
2
Z A 1 B C 3 D a
(1)  4    = u (1)  (1) (2t  (1))
Z A 3 B C 2 D 2
∆Z ∆A 1 ∆B ∆C 3 ∆D This (1) also have dimension of time
(2) =4 + – –
Z A 3 B C 2 D u(t) = LT–1 × T = L

∆Z ∆A 1 ∆B ∆C 3 ∆D a
(3) =4 + + – = (1) (2t - a ) = LT–2 × T × T = L
Z A 3 B C 2 D 2
Therefore, option (3) is the correct answer.
∆Z ∆A 1 ∆B ∆C 3 ∆D
(4) =4 + – +
Z A 3 B C 2 D 12. A physical quantity is given by X = [MaLbTc]. The percentage
1 error in measurements of M, L and T are α, β, γ. Then,
A4B 3
the maximum % error in the quantity X is:
Sol. Z  3
2
CD
(1) aα + bβ + c γ (2) aα + bβ − c γ
Z A 1 B C 3 D
 4    a b c
Z A 3 B C 2 D (3) + + (4) None of these
α β γ
Therefore, option (1) is the correct answer.
Sol. X = M a LbT c
9. In an experiment four quantities a, b, c and d are measured
with percentage error 1%, 2%, 3% and 4% respectively. dX dM dL dT
 a b c
a 3b2 X M L T
Quantity P is calculated as follows: P = . The percentage
cd dX
error in P is:  a  b  c
X
(1) 4 % (2) 14 % Therefore, option (1) is the correct answer.
(3) 10 % (4) 7 %
13. The absolute error in density of a sphere of radius 10.01 m
a 3b2 P  a b c d  and mass 4.692 kg is:
Sol. P    3 2   
cd P  a b c d  (1) 3.59 kg m–3 (2) 4.692 kg m–3
   3  1  2  2  3  4   14% (3) 0 (4) 1.12 kg m–3
Therefore, option (2) is the correct answer. m 4.692 × 3
Sol.
= fρ
=
10. The respective number of significant figure for the number 4 2 4 × 3.14 × ( 10.01 )3 × 10 −6
πr
23.023, 0.0003 and 2.1 × 10–3 are: 3
(1) 5, 1, 2 (2) 5, 1, 5 ρ = 1.12 × 103 kg /m3

(3) 5, 5, 2 (4) 4, 4, 2 abs. errors:


Δm = 1gm = 0.001 kg
Sol. 23.023 → 5, 0.0003 = 1, 2.1 × 10–3 = 2
Therefore, option (1) is the correct answer. Δr = 0.01 m
 m r
1  3
11. St =u + a ( 2t − 1) is:  m r
2
(1) Only numerically correct ∆P  0.001 3 × 0.01  3
∆ρ=  +  × 1.12 × 10
(2) Only dimensionally correct  4.692 10.01 
(3) Both numerically and dimensionally correct = 3.59 kg/m3
(4) Neither numerically nor dimensionally correct Therefore, option (1) is the correct answer.

Units and Measurements 25


14. The dimension of Planck’s constant equals to that of: 16. Dimensions of resistance in an electrical circuit, in terms of
(1) Energy (2) Momentum dimension of mass M, length L, time T and current I, would be:
(1) [ML2T-2] (2) [ML2T-1A-1]
(3) Angular momentum (4) Power
(3) [ML2T-3I-2] (4) [ML2T-3A-1]
Energy
Sol. Dimensions of Planck’s constant, h = V
Frequency Sol.  V = RI or R =
I
= [ML2T-1]
W  ML T 
2 2

Dimensions of angular momentum L Dimensions of V  


q  IT 
= Moment of inertia I × Angular velocity = [ML2][T-1]
 ML2T 2 / IT 
Therefore, option (3) is the correct answer. 
 R     ML2T 3A 2 
 I
15. In the following dimensionally correct equations, we have
Therefore, option (3) is the correct answer.
X
= F + Y, where F = force. The dimensional
Linear density 17. If A = (1.0 ± 0.2) m and B = (2.0 ± 0.2) m , then AB is:
formula of X are Y are: (1) 1.4 m ± 0.4 m (2) 1.41 m ± 0.15 m
(1) [M L T–2], [M2 L0 T–2] (3) 1.4 m ± 0.3 m (4) 1.4 m ± 0.2 m
Sol. x  AB  1.0  2.0  1.414 m
(2) [M2 L0 T–2], [M L T–2]
Rounding off x = 1.4 m
(3) [M L2 T–4], [M2 L–2 T–2]
x 1 A B  1  0.2 0.2 
      m  (in rounded figure)
(4) None of these x 2 A B  2  1.0 2.0 
 X  1
[F ] 
Sol.=  + [Y ] Dx = x   (0.3)

 L.D  2
∴ [Y] = [F] = [MLT–2]
1.4  0.3

Dx =  21
 X  2
 MLT   
2
1 
 X   M 2 L0 T 2 
 ML   AB  1.4  0.2  m Dx = 0.2 (after one decimal)
Therefore, option (2) is the correct answer. Therefore, option (4) is the correct answer.

26 P NEET (XI) Module-1 PHYSICS


W
SCHOOL LEVEL PROBLEMS

SINGLE CORRECT TYPE QUESTIONS 2. Assertion (A): Dimensional constants are the quantities
whose values are constant.
1. Which among the following is the supplementary unit____
Reason (R): Dimensional constants are dimensionless.
(1) Mass (2) Time
(1) Assertion and reason both are true and reason is correct
(3) Solid angle (4) Luminosity explanation for assertion.
2. The number of significant digits in 1559.00 is_____ (2) Assertion and reason both are true but reason is not
(1) 6 (2) 5 correct explanation for assertion.
(3) 3 (4) 4 (3) Assertion is true but reason is false.
(4) Assertion is false but reason is true.
3. Joule second is the unit of
3. Assertion (A): The method of dimensions analysis cannot
(1) Force (2) Angular momentum validate the exact relationship between physical quantities
(3) Energy (4) Power in any equation.
4. What is the number 75.66852 rounded off to 5 significant Reason (R): It does not distinguish between the physical
digits? quantities having same dimensions.
(1) 75.67 (2) 75.669 (3) 75.668 (4) 75.667 (1) Assertion and reason both are true and reason is correct
explanation for assertion.
5. The division of energy by time is X. The dimensional formula
(2) Assertion and reason both are true but reason is not
of X is same as that of
correct explanation for assertion.
(1) Power (2) Electric field (3) Assertion is true but reason is false.
(3) Momentum (4) Torque (4) Assertion is false but reason is true.
6. 1 kWh =
(1) 1000 W (2) 36 × 105 J VERY SHORT ANSWER TYPE QUESTIONS
(3) 1000 J (4) 3600 J 1. What is the number of significant figures in 0.06070?
7. Density of wood is 0.5 gm/cc in the CGS system of units. 2. If x = a + bt + ct2, where x is in meter and t in seconds, what
The corresponding value in MKS units is: is the unit of c?
(1) 500 (2) 5 3. Will the dimensions of a physical quantity be the same,
(3) 0.5 (4) 5000 whatever be the units in which it is measured? Why?
8. Which of the following is a derived unit? 4. What do you mean by the term measurement?
(1) Unit of mass (2) Unit of area 5. Define dimensions of a physical quantity.
(3) Unit of time (4) Unit of current 6. S.I units are coherent. Explain.
9. Newton/meter is the unit of:
2 7. What are the limitations of dimensional analysis? (Any two)
(1) Energy (2) Momentum
(3) Force (4) Pressure SHORT ANSWER TYPE QUESTIONS
10. Dimensional formula of heat energy is 1. 5.74 g of a substance occupies 1.2 cm3. Express its density
to correct significant figures.
(1) ML2T–2 (2) MLT–1
2. Derive the dimensional formula of:
(3) M0L0T–2 (4) None of these
(1) Angular velocity (2) Angular momentum
ASSERTION AND REASON TYPE QUESTIONS 3. If the velocity of light is taken as the unit of velocity and
one year as the unit of time, what must be the unit of length?
1. Assertion (A): If the units of force and length are doubled, What is it called?
the unit of energy will become four times.
4. For the determination of ‘g’ using a simple pendulum,
Reason (R): The unit of energy is independent of the units measurements of l and T are required. Error in the
of force and length. measurement of which of these will have larger effect on
(1) Assertion (A) and Reason (R) both are true and Reason the value of ‘g’ thus obtained and why? What is done to
(R) is correct explanation for Assertion (A). minimize this error?
(2) Assertion (A) and Reason (R) both are true but Reason 5. A gas bubble, from an explosion under water, oscillates
(R) is not correct explanation for Assertion (A). with a period T proportional to pa db Ec, where p is the static
(3) Assertion (A) is true but Reason (R) is false. pressure, d is the density of water and E is the total energy
(4) Assertion (A) is false but Reason (R) is true. of the explosion. Find the values of a, b and c.

Units and Measurements 27


6. The heat dissipated in a resistance can be determined from (iii) Which of the following sets have different dimensions
the relation: (1) Pressure and Young’s modulus
I 2 Rt (2) Emf and potential difference

H= cal
4.2 (3) Heat and work done
If the maximum errors in the measurement of current, (4) Dipole moment and electric flux
resistance and time are 2%, 1% and 1% respectively, what (iv) The dimension of a × b in the relations:
would be the maximum error in the dissipated heat?
b − x2
7. The length, breadth and thickness of a rectangular sheet of
P= , where P is power, x is distance and t is time.
at
metal are 4.234 m, 1.005 m and 2.01 cm, respectively. Give
the area and volume of the sheet to correct significant figures. (1) [M–1L2T2] (2) [M–1L0T2]
(3) [M L T ]
0 2 0
(4) [ML2T–1]
LONG ANSWER TYPE QUESTIONS (v) Find the value of 100 J on a system which has 20 cm, 250 g
1. The length, breadth and thickness of a rectangular sheet and half minute as fundamental units of length, mass and
of metal are 4.234 m, 1.005 m, and 2.01 cm, respectively. time.
Give the area and volume of the sheet to correct significant (1) 9 × 10–6 new units (2) 2.16 × 106 new units
figures. (3) 9 × 10 new units
6
(4) 100 new units
2. The frequency of vibration of a string depends on, (i) tension a 

in the string (ii) mass per unit length of string, (iii) vibrating 2. The Van der walls equation is  P + 2  (V − b) = RT Where
length of the string. Establish dimensionally the relation for  V 
frequency. P is pressure, V is volume, T is absolute temperature of given
sample of a gas, R is called molar gas constant, a and b are
CASE STUDY BASED QUESTIONS Vander waal’s constant. Now answer the following:
(i) The dimensional formula for b is same as for
1. The nature of physical quantity is described by its dimension.
All the physical quantities can be expressed in terms of some (1) V (2) PV2 (3) RT (4) P
combination of seven fundamental units. The dimensions of (ii) The dimensional formula for a is same as for
a physical quantity are thus the powers to which the base (1) V2 (2) P (3) PV2 (4) RT
quantities are raised to represent that quantity.
ab
If a given physical quantity depends on ath power of mass, bth (iii) The dimensional formula of is
power of length, cth power of time etc., then its dimensions RT
are expressed as [Ma Lb Tc]. (1) [ML5T–2] (2) [M0L3T0]
(i) The dimension of planck’s constant equal to that of (3) [ML–1T–2] (4) [M0L6T0]
(1) Energy (2) Momentum (iv) The dimensional formula for RT is same as for
(3) angular momentum (4) power (1) Energy (2) force
(ii) If force (F), velocity (V) and time (T) are taken as (3) Latent heat (4) Specific heat
fundamental units, then dimension of mass are (v) The dimensional formula for RT is not same as that for
(1) [FVT–2] (2) [FV–1T–1] ab a
(1) (2) Pb (3) (4) PV
(3) [FV–1T] (4) [FVT–1] V2 V2

28 P NEET (XI) Module-1 PHYSICS


W
PRARAMBH EXERCISE-1 (TOPICWISE)
DIMENSIONS & DIMENSIONAL FORMULA 9. The dimensions of intensity are:
(1) M1 L0 T–3 (2) M2 L1 T–2
1. Select the pair whose dimensions are same.
(1) Pressure and stress (2) Momentum and impulse (3) M1 L2 T–2 (4) M2 L0 T–3
(3) Torque and energy (4) All of these 10. Surface tension has the same dimensions as that of:
2. The velocity “v” of a particle is given in terms of time t as (1) Coefficient of viscosity
b (2) Impulse
v = at + .
t +c (3) Momentum
The dimensions of a, b, c are: (4) Spring constant
(1) L2 ; ML ; T–2 (2) LT2 ; LT ; L 11. Which one of the following has the dimensions of pressure?
(3) LT–2 ; L ; T (4) L ; LT ; T2 ML M
(1) (2)
3. Which of the following pairs of physical quantities does not T2 L2 T 2
have same dimensional formula? M M
(3) (4)
(1) Work and torque LT 2 LT
(2) Angular momentum and Planck’s constant 12. The dimensional formula of wave number is:
(3) Tension and surface tension
(1) [M0L0T–1] (2) [M–1L–1T0]
(4) Impulse and linear momentum
(3) [M0L–1T0] (4) [M0L0T0]
4. The equation of a wave is given by:
13. Dimensions of gravitational constant are:
x 
y= A sin ω  − k  (1) M–1L3T–2 (2) M–2L3T–1
 v 
(3) M3L–1T–2 (4) M–1L2T–3

where ω is the angular velocity and v is the linear velocity.
The dimension of k is: 14. The dimensional formula of angular velocity is:
(1) LT (2) T (1) [M0L0T–1] (2) [MLT–1]
(3) T–1 (4) T2 (3) [M0L0T1] (4) [ML0T–2]
5. A unitless quantity: 15. The dimensions of planck’s constant and angular momentum
(1) May have a non-zero dimension are respectively:
(2) Always has a non-zero dimension (1) M1L2T–1 and M1LT–1 (2) M1L2T–1 and M1L2T–2
(3) Never has a non-zero dimension
(3) M1LT–1 and M1L2T–1 (4) M1L2T–1 and M1L2T–1
(4) Does not exist
16. The pair having the same dimensions are:
6. The Young’s modulus of steel is 1.9 × 1011 N/m2. When
(1) Angular momentum, work
expressed in CGS units of dyne/cm2, it will be equal to
(1 N = 105 dyne, 1 m2 = 104 cm2) (2) Work, torque
(3) Potential energy, linear momentum
(1) 1.9 × 1010 (2) 1.9 × 1011
(4) Kinetic energy, velocity
(3) 1.9 × 1012 (4) 1.9 × 1013
17. Dimensional formula of DQ, heat supplied to the system is
7. A dimensionless quantity:
given by:
(1) May have a unit (2) Never has a unit
(1) M1L2T–2 (2) M1L1T–2
(3) Always has a unit (4) Doesn’t exist
(3) M1L2T–1 (4) ML1T–1
8. The position x of a particle at time “t” is given by-
18. [ML2T–3] is the dimension of:
v
x = 0 (1- e-at ) (1) Work (2) Power
a

Where v0 is a constant and a > 0. (3) Force (4) Momentum

The dimensions of v0 and a are: 19. Which of the following ratios express pressure?
(1) M0 L T–1 ­and T–1 (2) M0 L T0 ­and T–1 (1) Force/Length (2) Energy/Volume
(3) M0 L T–1 ­and LT–2 (4) M0 L T–1 ­and T (3) Energy/Area (4) Force/Volume

Units and Measurements 29


20. The dimensions of force constant of spring are: dy
31. In the relation: = 2ωsin ( ωt + φ0 ) , the dimensional
(1) MT –2
(2) MLT –1
dt
(3) MLT –2
(4) ML–1 T–1 formula for (ωt + ϕ0) is:
(1) [MLT] (2) [MLT0]
21. The dimensions of emf in MKS system of unit is:
(3) [ML0T0] (4) [M0L0T0]
(1) ML–1T–2Q–2 (2) ML2T–2Q–2
32. Dimensions of Stefan’s constant are:
(3) MLT–2Q–1 (4) ML2T–2Q–1
(1) [M L2 T–2] (2) [ML2 T–2 θ–4]
22. The dimensions of physical quantity X in the equation force
(3) [M T–3 θ–4] (4) [M L0 T–2]
X
= is given by: 33. The dimension of angular momentum is:
Density
(1) [ML2 T–2] (2) [ML–2 T–1]
(1) M L T
1 4 –2
(2) M L T
2 –2 –1
(3) [MLT–1] (4) [ML2 T–1]
3 −1
(3) M 2 L 2
T −2 (4) M1L–2T–1 1
34. The dimensions of “K” in equation W = Kx 2 is:
23. Given that V is speed, r is the radius and g is the acceleration 2
due to gravity. Which of the following is dimensionless? (1) [M1 L0 T–2] (2) [M0 L1 T–1]
(1) V2/rg (2) V2r/g (3) [M1 L1 T–2] (4) [M1 L0 T–1]
(3) V2g/r (4) V2rg h has the dimension of:
35.
24. Which of the following is not a unit of time? 2π
(1) Second (2) Minute (1) Velocity (2) Momentum
(3) Year (4) Light year (3) Energy (4) Angular momentum
25. The dimensions of potential are the same as that of:
(1) Work APPLICATIONS OF DIMENSIONAL ANALYSIS
(2) Electric field per unit charge 36. In a particular system, the unit of length, mass and time are
(3) Work per unit charge chosen to be 10cm, 10g and 0.1 sec respectively. The unit
(4) Force per unit charge of force in this system will be equivalent to:
26. The dimensions formula for latent heat is: (1) 0.1N (2) 1N
(1) [M L T ]
0 2 –2
(2) [MLT ] –2 (3) 10N (4) 100N

(3) [ML2T–2] (4) [ML2T–1] 37. If momentum (p), area (A) and time (T) are taken to be
fundamental quantities, then energy has the dimensional
formula:
γP
27. If V = , then dimensions of γ are:
ρ (1) pA–1T1 (2) p2AT
(3) pA–1/2T (4) pA1/2T-1
(1) [M0L0T0] (2) [M0L0T–1]
38. If units of length, mass and force are chosen as fundamental
(3) [M1L0T0] (4) [M0L1T0] units, the dimensions of time would be:
28. In the relation: y = a sin (ωt – kx), (1) M1/2 L–1/2 F1/2 (2) M1/2 L1/2 F1/2

the dimensional formula for k is: (3) M1/2 L1/2 F–1/2 (4) M1 L–1/2 F–1/2
(1) [M0LT] (2) [M0L–1T0]
39. If speed of light (c), acceleration due to gravity (g) and
(3) [M0LT–1] (4) [M0L–1T–1] pressure (P) are taken as fundamental units, the dimensions
29. In the relation y = a cos (ωt + kx), the dimensional formula for of gravitational constant (G) are:
kx is same as that of: (1) c0 g P–3 (2) c2 g3 P–2
(1) a/ω (2) a/y (3) c0 g2 P–1 (4) c2 g2 P–2
(3) ωt/a (4) ya/ωt 40. If energy (E), velocity (V) and force (F) be taken as
30. The modulus of elasticity is dimensionally equivalent to: fundamental quantity, then what are the dimensions of mass:
(1) Strain (2) Force (1) EV2 (2) EV–2
(3) Stress (4) Coefficient of viscosity (3) FV–1 (4) FV–2

30 P NEET (XI) Module-1 PHYSICS


W
41. If area (A), velocity (V) and density (ρ) are taken as 49. Measure of two quantities along with the precision of
fundamental units, what is the dimensional formula for respective measuring instrument is:
force? A = 2.5 ms–1 ± 0.5 ms–1, B = 0.10 s ± 0.01 s. The value of
(1) [AV ρ]
2
(2) [A Vρ] 2 AB will be:
(3) [AVρ2] (4) [AVρ] (1) (0.25 ± 0.08) m (2) (0.25 ± 0.5) m
42. A student writes the escape velocity as: (3) (0.25 ± 0.05) m (4) (0.25 ± 0.135) m

2GM 50. A wire has a mass 0.3 ± 0.003g , radius 0.5 ± 0.005 mm and
ve = length 6 ± 0.06 cm. The maximum percentage error in the
R
measurement of density is:

The equation is:
(1) 1% (2) 2%
(1) Dimensionally incorrect
(3) 3% (4) 4%
(2) Dimensionally correct
51. The values of two resistors are R1 = (6 ± 0.3) kΩ and
(3) Numerically correct R2 = (10 ± 0.2) kΩ. The percentage error in the equivalent
(4) Both (2) and (3) resistance when they are connected in parallel is:
(1) 5.125% (2) 2%
ERRORS & MEASUREMENT, SIGNIFICANT (3) 3.125% (4) 10.125%
FIGURES 52. Which of the following measurement is most precise?
43. The number of significant figures in 0.06900 is: (1) 5.00 mm (2) 5.00 cm
(1) 5 (2) 4 (3) 5.00 m (4) 5.00 km
(3) 2 (4) 3 ∆x
53. If x = an , then fractional error is equal to:
44. If the length of rod A is 3.25 ± 0.01cm and that of rod B is x
4.19 ± 0.01cm, then the rod B is longer than rod A by:
 a   a 
n

(1) 0.94 ± 0.00 cm (2) 0.94 ± 0.01 cm (1)    (2)  n  


 a   a 
(3) 0.94 ± 0.02 cm (4) 0.94 ± 0.005 cm
a a
45. The sum of the numbers 436.32, 227.2 and 0.301 in (3) n log e (4) n log
a a
appropriate significant figures is:
(1) 663.821 (2) 664 54. If voltage V = (100 ± 5)V and current I = (10 ± 0.2)A, the
percentage error in resistance R is:
(3) 663.8 (4) 663.82
(1) 5.2% (2) 25%
46. A physical quantity A is related to four observations
(3) 7% (4) 10%
a 2 b3
a, b, c and d as follows, A = . The percentage errors 55. The mean length of an object is 5 cm. Which of the following
c d
measurements is most accurate?
of measurement in a, b, c and d are 1%, 3%, 2% and 2%
(1) 4.9 cm (2) 4.805 cm
respectively. What is the percentage error in the measurement
(3) 5.25 cm (4) 5.4 cm
of quantity A?
(1) 12% (2) 7%
MEASURING INSTRUMENTS
(3) 5% (4) 14% 56. In an experiment, it is required to measure the distance
47. The numbers 2.745 and 2.735 on rounding off to 3 significant between two points which are between 0.7 m to 0.8 m
figures will give: apart. Which of the following instruments should be used
(1) 2.75 and 2.74 (2) 2.74 and 2.73 so that the distance can be measured to within an accuracy
of 0.001 m?
(3) 2.75 and 2.73 (4) 2.74 and 2.74
(1) A metre rule and a pair of vernier calipers
48. Error in the measurement of radius of a sphere is 1%. The
error in the calculated value of its volume is: (2) A half-metre rule
(1) 1% (2) 3% (3) A metre rule
(3) 5% (4) 7% (4) A ten-metre measuring tape

Units and Measurements 31


57. The figure below shows the final reading on a vernier (1) 0.5 mm (2) 0.1 mm
caliper with a zero error of -0.3 mm. What is the actual (3) 0.05 mm (4) 0.005 mm
measurement?
62. A vernier caliper has 20 divisions on the vernier scale, which
10 cm 11 cm coincide with 19 on the main scale. The least count of the
instrument is 0.1 mm. The main scale divisions are of–
(1) 0.5 mm (2) 1 mm
(3) 2 mm (4) 1/4 mm
0 5 10
63. A vernier caliper having 1 main scale division = 0.1 cm is
(1) 10.32 cm (2) 9.99 cm
designed to have a least count of 0.02 cm. If n be the number
(3) 10.05 cm (4) 9.72 cm of divisions on vernier scale and m be the length of vernier
58. A workman measures, as accurately as possible, the length scale, then
and internal diameter of a straight copper pipe. The length (1) n = 10, m = 0.5 cm
is approximately 600 cm and the internal diameter is
(2) n = 9, m = 0.4 cm
approximately 2 cm.
(3) n = 10, m = 0.8 cm

What is the best combination of instruments for the workman
(4) n = 10, m = 0.2 cm
to use?
64. The dimensions of a rectangular block measured
Internal Diameter Length
with a vernier caliper having least count of 0.1 mm is
A. Ruler Ruler 5 mm × 10 mm × 5 mm. The maximum percentage error in
B. Ruler Tape measurement of volume of the block is
C. Vernier Calipers Ruler (1) 5 % (2) 10 %
D. Vernier Calipers Tape (3) 15 % (4) 20 %

(1) A (2) D 65. The pitch of a screw gauge is 0.5 mm and there are 100
divisions on it circular scale. The instrument reads +2
(3) B (4) C
divisions when nothing is put in-between its jaws. In
59. In a vernier calipers, one main scale division is x cm and measuring the diameter of a wire, there are 8 divisions on
n division of the vernier scale coincide with (n-1) divisions of the main scale and 83rd division coincides with the reference
the main scale. The least count (in cm) of the calipers is: line. Then the diameter of the wire is

(2) 
nx 
(1)  n  1  x 
(1) 4.05 mm (2) 4.405 mm
 n   n 1  (3) 3.05 mm (4) 1.25 mm
66. In an experiment four quantities a, b, c and d are measured
(3) x (4)  x 
n  n 1  with percentage error 1%, 2%, 3% and 4% respectively.
a 3b 2
60. A student measure the diameter of a thick wire using a screw Quantity P is calculated as follows P = . % error in P is:
cd
gauge of least count 0.001 cm. The main scale reading is
2 mm and zero of circular scale division coincides with (1) 4% (2) 14%
50 division above the reference level. If the screw gauge has (3) 10% (4) 7%
a zero error of 0.002 cm, the correct diameter of the thick
67. The smallest division on the main scale of a vernier caliper
wire in cm is:
is 1 mm, and 10 vernier divisions coincide with 9 main scale
(1) 0.248 (2) 0.428 divisions. While measuring the diameter of a sphere, the
(3) 0.521 (4) 0.224 zero mark of the vernier scale lies between 2.0 and 2.1 cm
61. In a vernier caliper, ten smallest divisions of the vernier scale and the fifth division of the vernier scale coincide with a
are equal to nine smallest division on the main scale. If the scale division. Then diameter of the sphere is
smallest division on the main scale is half millimeter, then (1) 2.05 cm (2) 3.05 cm
the vernier constant is: (3) 2.50 cm (4) None of these

32 P NEET (XI) Module-1 PHYSICS


W
PRABAL EXERCISE-2 (LEARNING PLUS)
1. The time dependence of a physical quantity P is given by
Xn
P = P0 exp (–αt2), where α is a constant and t is time. The 8. If Q = and ∆x is absolute error in the measurement of X,
Ym
constant α is: ∆y is absolute error in the measurement of Y, then absolute
(1) Dimensionless (2) Dimensions T–2 error ∆Q in Q is:
(3) Dimension of P (4) Dimensions T2
 x y 
(1) Q    n m 
2. Which of the following sets cannot enter into the list of  x y 
fundamental quantities in any system of units?
 x y 
(1) Length, time and velocity (2) Q    n m Q
 x y 
(2) Length, mass and velocity
(3) Mass, time and velocity  x y 
(3) Q    n m Q
(4) Length, time and mass  x y 

3. The velocity of a particle (v) at a instant (t) is given by  mx ny 


(4) Q     Q
v = at + bt2 the dimension of b is:  x y 
(1) L (2) LT–1 9. The length of a rod is (11.05 ± 0.05)cm. What is the length
(3) LT
–2
(4) LT –3 of two rods?

4. If force (F), length (L) and Current (I) and time (T) are taken (1) (22.1 ± 0.05) cm (2) (22.1 ± 0.2) cm
as bases then the dimensions of ε0 are: (3) (22.10 ± 0.05) cm (4) (22.10 ± 0.10) cm
(1) [FL2 I2 T–2] (2) [F–1 L2 I2 T2] 10. A uniform wire of length L, diameter D and density ρ is
stretched under a tension T. The correct relation between
(3) [F–1L–2T2I2] (4) [F2L2T2I2]
its fundamental frequency f, the length L and the diameter
5. The SI unit of energy is J = kg m2 s–2 that of speed v is D is:
ms–1 and of acceleration a is ms–2. Which of the formula 1 1
for kinetic energy (K) given below can you rule out on the (1) f ∝ (2) f ∝
LD L D
basis of dimensional arguments (m stands for the mass of
the body). (3) f ∝
1
(4) f ∝
1
D2 LD 2
I. K = m2v2 II. K = (1/2) mv2
11. If E = energy, G = gravitational constant, I = Impulse and
III. K = ma IV. K = (3/16) mv2
GIM 2
M = mass, then dimensions of are same as that of
1 E2
V. K   mv 2 + ma
=
2 (1) Time (2) Mass
(1) I and II (2) Only II (3) Length (4) Force
(3) II and IV (4) I, III and V 12. A gas bubble formed from an explosion under water
oscillates with a period T proportional to pa db Ec, where p
A 3 b1/ 2
6. A physical quantity P is given by P = . The quantity is pressure, d is the density of water and E is the total energy
C-4 D3/ 2
of explosion. The values of a, b and c are
which brings in the maximum percentage error in P is:
(1) a = 1, b = 1, c = 2 (2) a = 1, b = 2, c = 1
(1) A (2) B (3) C (4) D
5 1 1 5 1 1
7. A body travels uniformly a distance of (13.8 ± 0.2)m in a (3) =a =,b ,c = (4) a   , b  , c 
6 2 3 6 2 3
time (4.0 ± 0.3)sec. The velocity of the body within error
limits is: 13. A normal human eye can see an object making an angle of
1.8° at the eye. What is the approximate height of object
(1) (3.45 ± 0.2)m/sec (2) (3.45 ± 0.3)m/sec
which can be seen by an eye placed at a distance of 1 m
(3) (3.45 ± 0.4)m/sec (4) (3.45 ± 0.5)m/sec from the object?

Units and Measurements 33


20. A liquid of coefficient of viscosity η is flowing steadily
h 0 through a capillary tube of radius “r” and length “l”. If V is
1.8
volume of liquid flowing per second, the pressure difference
1m
“P” at the end of tube is given by:
(1) π cm (2) 2π cm
(3) 4π cm (4) 3π cm 8πlV 8ηr 4 l
(1) P = (2) P =
ηr 4 πV
z
α −α
14. In the relation P = e Kθ P is pressure, Z is the distance, 8ηlV
β 8ηr 4 V
(3) P = (4) P =
K is Boltzmann’s constant and θ is the temperature. The πr 4 πl
dimensional formula of α will be:
21. The mass and volume of a body are 4.237 g and 2.5 cm3,
(1) [M L T ]
1 1 –2
(2) [M L T ] 1 2 1
respectively. The density of the material of the body in
(3) [M L T ]
1 0 –1
(4) [M0L2T–1] correct significant figures is:
(1) 1.6048 g cm–3 (2) 1.69 g cm–3
d    
15. C o n s i d e r t h e e q u a t i o n (
dt ∫ ) ( )
F·dS = A F·p w h e r e (3) 1.7 g cm–3 (4) 1.695 g cm–3
  
F = force, s = displacement, t = time and p = momentum. 22. The length and breadth of a rectangular sheet are 16.2 cm and
The dimensional formula of A will be: 10.1 cm, respectively. The area of the sheet in appropriate
significant figures and error is:
(1) M0L0T0 (2) ML0T0
(3) M–1L0T0 (4) M0L0T–1 (1) 164 ± 3 cm2

16. If P, Q, R are physical quantities, having different (2) 163.62 ± 2.6 cm2
dimensions, which of the following combinations can never (3) 163.6 ± 2.6 cm2
be a meaningful quantity?
(4) 163.62 ± 3 cm2
(1) (P – Q) (2) PQ – R
R 23. You measure two quantities as A = (1.0 ± 0.2 m), B = (2 .0 m
2
PQ (PR − Q ) ± 0.2 m). We should report correct value for AB as:
(3) (4)
R R
(1) 1.4 m ± 0.4 m
17. Given that the displacement of an oscillating particle is
given by y = A sin(Bx + Ct + D). The dimensional formula (2) 1.41 m ± 0.15 m
for (ABCD) is: (3) 1.4 m ± 0.3 m
(1) [M0L–1T0] (2) [M0L0T–1]
(4) 1.4 m ± 0.2 m
(3) [M0L–1T–1] (4) [M0L0T0]
24. On the basis of dimensions, decide which of the following
α
18. Force F and density d are related as F = then find relations for the displacement of a particle undergoing simple
β+ d harmonic motion is not correct?
the dimensions of α:
(1) [M1/2L–½T–2] (2) [M3/2L½T2] (1) y = a sin 2π t/T
(3) [M3/2L–½T–2] (4) [M2L–½T2] vt
(2) y = a sin
19. Frequency is the function of density (ρ), length (a) and λ
surface tension (T). The value is:
a t
kρ1/ 2 a 3/ 2 kρ3/ 2 a 3/ 2 (3) y = sin  
(1) (2) t a
T T
 2 πt 2 πt 
kρ1/ 2 a 3/ 2 (4) y a 2  sin
= − cos 
(3) (4) None of these  T T 
T 3/ 4

34 P NEET (XI) Module-1 PHYSICS


W
PARIKSHIT EXERCISE-3 (MULTICONCEPT)
MATCH THE COLUMN MCQs (1) A-(P); B-(P); C-(S); D-(R)

1. There are four Vernier scales, whose specifications are (2) A-(P); B-(Q); C-(R); D-(S)
given in column-I and the least count is given in Column-II. (3) A-(R); B-(P); C-(Q); D-(S)
Match the Columns-I and II with correct specification and (4) None of these
corresponding least count (s = value of main scale division,
4. Some physical quantities are given in Column-I and some
n = number of marks on Vernier). Assume (n–1) main scale
possible SI Units in which these quantities may be expressed
divisions are equal to n Vernier divisions.
are given in Column-II. Match the physical quantities in
Column-I Column-II Column-I with the units in Column-II.

A. s = 1 mm, n = 10 P. 0.05 mm Column-I Column-II


B. s = 0.5 mm, n = 10 Q. 0.01 mm A. GMeMs P. (volt)(coulomb)(metre)
C. s = 0.5 mm, n = 20 R. 0.1 mm B. 3RT Q. (kilogram)
M (metre)3(second)–2
D. s = 1 mm, n = 100 S. 0.025 mm

(1) A-(Q); B-(R); C-(P); D-(S) F2


C. R. (meter)2(second)–2
q 2 B2
(2) A-(R); B-(P); C-(S); D-(Q)

(3) A-(P); B-(Q); C-(R); D-(S) GM e


D. S. (farad)(volt)2(kg)–1
Re
(4) None of these
2. Match the columns. Where G is universal gravitational constant; Me, mass of the
earth; Ms, mass of sun; Re, radius of the earth R, universal
Column-I Column-II gas constant; T, absolute temperature; M, molar mass; F,
A. Backlash error P. Always subtracted force; q charge; B, magnetic field.
B. Zero error Q. Least count = 1 MSD – (1) A-(P, Q); B-(R, S); C-(R, S); D-(R, S)
1 VSD
(2) A-(R, S); B-(P, Q); C-(Q, R); D-(S, P)
May be negative or
C. Vernier callipers R. (3) A-(R, S); B-(P, S); C-(S, P); D-(S, Q)
positive
Error in screw (4) A-(Q, S); B-(P, S); C-(S, P); D-(S, R)
D. S. Due to loose fittings
gauge 5. Match Column-I with Column-II and select the correct
answer using the codes given below the list:
(1) A-(S); B-(P, R); C-(Q); D-(R, S)

(2) A-(P); B-(Q); C-(R); D-(S) Column-I Column-II

(3) A-(S); B-(P); C-(Q, R); D-(S) A. Boltzmann constant P. [ML2T–1­]

(4) None of these B. Coefficient of viscosity Q. [ML–1T–1]


3. Using significant figures, match the following. C. Planck’s constant R. [MLT–3K–1]
Column-I Column-II D. Thermal conductivity S. [ML2T–2K–1]
A. 0.12345 P. 5
(1) A-(R); B-(P); C-(Q); D-(S)
B. 0.12100 cm Q. 4
(2) A-(R); B-(Q); C-(P); D-(S)
C. 47.23 ÷ 2.3 R. 1
(3) A-(S); B-(Q); C-(P); D-(R)
D. 3 × 108 S. 2
(4) A-(S); B-(P); C-(Q); D-(R)

Units and Measurements 35


STATEMENT BASED MCQs ASSERTION & REASON MCQs
(1) Both Statement-I and Statement-II are correct. (1) Assertion (A) is true, Reason (R) is true; Reason (R) is a
correct explanation for Assertion (A).
(2) Both Statement-I and Statement-II are incorrect.
(2) Assertion (A) is true, Reason (R) is true; Reason (R) is
(3) Statement-I is correct & Statement-II is incorrect. not a correct explanation for Assertion (A).
(4) Statement-I is incorrect & Statement-II is correct. (3) Assertion (A) is true, Reason (R) is false.
(4) Assertion (A) is false, Reason (R) is true.
6. Statement-I: When percentage errors in the measurement
of mass and velocity are 1% and 2% respectively, the 12. Assertion (A): Mass, volume and time may be taken as
percentage error in KE is 5%. fundamental quantities in a system of units.

1 2 ∆E ∆m 2∆v Reason (R): Mass, volume and time are independent of



Statement-II: KE or=
E mv , = + . one another.
2 E m v
13. Assertion (A): Out of three measurements l = 0.7 m;
7. Statement-I: A screw gauge having a smaller value of pitch
l = 0.70 m and l = 0.700 m, the last one is most accurate.
has greater accuracy.
Reason (R): In every measurement, only the last significant

Statement-II: The least count of screw gauge is directly digit is not accurately known.
proportional to the number of divisions on circular scale. 14. Assertion (A): If x and y are the distance along x and y axes
8. Statement-I: Unit chosen for measuring physical quantities d3y
respectively then the dimensions of is M0L–2T0.
should not be accessible. dx 3
b
Statement-II: Unit should change with the changing Reason (R): Dimensions of
∫ ydx is M0L2T0.
physical conditions like temperature, pressure, etc. a

15. Assertion (A): When we change the unit of measurement


9. Statement-I: Specific gravity of a fluid is a dimensionless of a quantity, its numerical value changes.
quantity.
Reason (R): Smaller the unit of measurement smaller is its
Statement-II: It is the ratio of density of fluid to the density numerical value.
of water. 16. Assertion (A): Random errors arise due to random and
unpredictable fluctuations in experimental conditions.
10. Statement-I: Absolute error is unitless and dimensionless.
Reason (R): Random errors occurred due to irregularly with

Statement-II: All types of errors are unitless and respect to sign and size.
dimensionless. 17. Assertion (A): Special functions such as trigonometric,
11. Statement-I: Two physical quantities having same logarithmic and exponential functions are not dimensionless.
dimensions, may have different units. Reason (R): A pure number, ratio of similar physical

quantities, such as angle and refractive index, has no
Statement-II: Shake and light year, both measure time. dimensions.

36 P NEET (XI) Module-1 PHYSICS


W
PYQ’s
PYQ’s EXERCISE-4 (NEET PAST YEAR QUESTIONS)
1. The errors in the measurement which arise due to 8. A screw gauge has least count of 0.01 mm and there are
unpredictable fluctuations in temperature and voltage supply 50 divisions in its circular scale.
are: (2023)
The pitch of the screw gauge is: (2020)
(1) Personal errors (1) 0.25 mm (2) 0.5 mm
(2) Least count errors (3) 1.0 mm (4) 0.01 mm
9. Taking into account of the significant figures, what is the
(3) Random errors
value of 9.99 m – 0.0099 m? (2020)
(4) Instrumental errors (1) 9.98 m (2) 9.980 m
2. A metal wire has mass (0.4 ± 0.002)g, radius (0.3 ± 0.001) mm (3) 9.9 m (4) 9.9801 m
and length (5 ± 0.02) cm. The maximum possible percentage 10. Dimensions of stress are: (2020)
error in the measurement of density will nearly be: (2023) (1) [ML T ]
2 –2
(2) [ML T ]
0 –2

(1) 1.3% (2) 1.6% (3) 1.4% (4) 1.2% (3) [ML T ]
–1 –2
(4) [MLT–2]
3. Plane angle and solid angle have: (2022) 11. The intervals measured by a clock given the following
readings:
(1) Both units and dimension
1.25 s, 1.24 s, 1.27 s, 1.21 s and 1.28 s. What is the
(2) Units but no dimensions percentage relative error is the observations?(2020 Covid)
(3) Dimensions but no units (1) 4% (2) 16%
(4) No units and no dimensions (3) 1.6% (4) 2%
12. The angle of 1′ (minute of arc) in radian is nearly equal to
4. The area of a rectangular field (in m2) of length 55.3 m
 (2020 Covid Re)
and breadth 25 m after rounding off the value for correct
(1) 4.85 × 10–4 rad (2) 4.80 × 10–6 rad
significant digits is: (2022)
(3) 1.75 × 10–2 rad (4) 2.91 × 10–4 rad
(1) 14 × 102 (2) 138 × 101
13. In an experiment, the percentage of error occurred in the
(3) 1382 (4) 1382.5 measurement of physical quantities A, B, C and D are 1%, 2%,
5. If E and G respectively denote energy and gravitational 3% and 4% respectively. Then the maximum percentage of error
1
constant, then E/G has the dimensions of:  (2021) A 2B 2
in the measurement of X, where X = 1
will be  (2019)
(1) [M][L ][T ]
–1 –1
(2) [M][L ][T ]
0 0 C 3 D3
3
(3) [M2][L–2][T–1] (4) [M2][L–1][T0] (1)   % (2) 16%
 13 
6. A screw gauge gives the following readings when used to (3) – 10% (4) 10%
measure the diameter of a wire  (2021) 14. A student measured the diameter of a small steel ball using a
Main scale reading : 0 mm screw gauge of least count 0.001 cm. The main scale reading
is 5 mm and zero of circular scale division coincides with
Circular scale reading : 52 divisions 25 divisions above the reference level. If screw gauge has a zero
Given that 1 mm on main scale corresponds to 100 divisions error of –0.004 cm, the correct diameter of the ball is (2018)
on the circular scale. The diameter of the wire from the above (1) 0.053 cm (2) 0.525 cm
data is: (3) 0.521 cm (4) 0.529 cm
(1) 0.026 cm (2) 0.26 cm 15. A student performs an experiment of measuring the thickness
of a slab with a vernier caliper whose 50 divisions of the
(3) 0.052 cm (4) 0.52 cm vernier scale are equal to 49 divisions of the main scale.
7. If force [F], acceleration [A] and time [T] are chosen as the He noted that zero of the vernier scale is between 7.00 cm
and 7.05 cm mark of the main scale and 23rd division of the
fundamental physical quantities. Find the dimensions of
vernier scale exactly coincides with the main scale. The
energy.  (2021)
measured value of the thickness of the given slab using the
(1) [F] [A] [T2] (2) [F] [A] [T–1] caliper will be: (2017-Gujarat)
(1) 7.73 cm (2) 7.23 cm
(3) [F] [A–1] [T] (4) [F] [A] [T] (3) 7.023 cm (4) 7.073 cm

Units and Measurements 37


16. Planck’s constant (h), speed of light in vacuum (c) and 18. If dimension of critical velocity of liquid flowing through
Newton’s gravitational constant (G) are three fundamental a tube are expressed as νc∝[ηxρyrz] where η, ρ and r are the
constants. Which of the following combinations of these has coefficient of viscosity of liquid, density of liquid and radius
the dimension of length? (2016-II) of the tube respectively, then the values of x, y and z are
given by: (2015 - Re)
hc Gc hG hG
(1) (2) (3) (4)  (1) 1, 1, 1 (2) 1, –1, –1
G h 3/ 2 c3/ 2 c5/ 2
(3) –1, –1, 1 (4) –1, –1, –1
17. If energy (E), velocity (V) and time (T) are chosen as the
fundamental quantities, the dimensional formula of surface 19. If Force (F), Velocity (V) and Time (T) are taken as
tension will be: (2015) fundamental units, then the dimensions of mass are: ( 2014)

(1) [EV–1T–2] (2) [EV–2T–2] (1) [F V T–1] (2) [F V T–2]


(3) [E–2V–1T–3] (4) [EV–2T–1] (3) [F V-1 T–1] (4) [F V–1T]

38 P NEET (XI) Module-1 PHYSICS


W
Answer Key

CONCEPT APPLICATION
ρr 3
1. [M2L–2T–2] 2. [T] 3. [M0L1T0] 4. K
5. x = 1; y = –1; z = 1 6. M = [V–2F0E] 7. [F0AT2]
S
8. [AV2ρ]. 9. [ML11/2T–2] 10. 10–3 N 11.2.16 × 1012 unit 12. (1) 13. (1) 14. (4) 15. (3) 16. (2)

17. (4) 18. (3) 19. (2) 20. (4) 21. (4) 22. (4) 23. (4) 24. (3) 25. (4)

26. (i) 0.005 mm (ii) 2.175 mm 27. (4)

SCHOOL LEVEL PROBLEMS


Single Correct Type Questions
1. (3) 2. (1) 3. (2) 4. (3) 5. (1) 6. (2) 7. (1) 8. (2) 9. (4) 10. (1)
Assertion and Reason Type Questions
1. (3) 2. (3) 3. (2)
Case Study Based Questions
1. (i) (3), (ii) (3), (iii) (4), (iv) (1), (v) (3) 2. (i) (1), (ii) (3), (iii) (4), (iv) (1), (v) (3)

PRARAMBH EXERCISE-1 (TOPICWISE)


1. (4) 2. (3) 3. (3) 4. (2) 5. (3) 6. (3) 7. (1) 8. (1) 9. (1) 10. (4)
11. (3) 12. (3) 13. (1) 14. (1) 15. (4) 16. (2) 17. (1) 18. (2) 19. (2) 20. (1)
21. (4) 22. (3) 23. (1) 24. (4) 25. (3) 26. (1) 27. (1) 28. (2) 29. (2) 30. (3)
31. (4) 32. (3) 33. (4) 34. (1) 35. (4) 36. (1) 37. (4) 38. (3) 39. (3) 40. (2)
41. (1) 42. (4) 43. (2) 44. (3) 45. (3) 46. (4) 47. (4) 48. (2) 49. (1) 50. (4)
51. (4) 52. (1) 53. (2) 54. (3) 55. (1) 56. (1) 57. (3) 58. (2) 59. (3) 60. (1)
61. (3) 62. (3) 63. (3) 64. (1) 65. (2) 66. (2) 67. (1)

PRABAL EXERCISE-2 (LEARNING PLUS)


1. (2) 2. (1) 3. (4) 4. (3) 5. (4) 6. (3) 7. (2) 8. (2) 9. (4) 10. (1)
11. (1) 12. (4) 13. (1) 14. (1) 15. (3) 16. (1) 17. (2) 18. (3) 19. (4) 20. (3)
21. (3) 22. (1) 23. (4) 24. (3)

PARIKSHIT EXERCISE-3 (MULTICONCEPT)


1. (2) 2. (1) 3. (1) 4. (1) 5. (3) 6. (1) 7. (3) 8. (2) 9. (1) 10. (2)
11. (3) 12. (1) 13. (2) 14. (2) 15. (3) 16. (2) 17. (4)

PYQ’s EXERCISE-4 (NEET PAST YEAR QUESTIONS)


1. (3) 2. (2) 3. (2) 4. (1) 5. (4) 6. (3) 7. (1) 8. (2) 9. (1) 10. (3)
11. (3) 12. (4) 13. (2) 14. (4) 15. (3) 16. (3) 17. (2) 18. (2) 19. (4)

Units and Measurements 39

You might also like